You are on page 1of 85

USMLE WORLD STEP 1

BIOCHEMISTRY
Question List

Biochemistry Q No:

Endocrinology

Biochemistry Q No:

43

Hepatobiliary system

Biochemistry Q No:

Neurology

Biochemistry Q No:

44

Neurology

Biochemistry Q No:

Neurology

Biochemistry Q No:

45

Neurology

Biochemistry Q No:

Endocrinology

Biochemistry Q No:

46

Endocrinology

Biochemistry Q No:

Endocrinology

Biochemistry Q No:

47

Gastrointestinal system

Biochemistry Q No:

Hepatobiliary system

Biochemistry Q No:

48

Endocrinology

Biochemistry Q No:

Hematology

Biochemistry Q No:

49

Endocrinology

Biochemistry Q No:

Endocrinology

Biochemistry Q No:

50

Hematology

Biochemistry Q No:

Hepatobiliary system

Biochemistry Q No:

51

Pulmonology

Biochemistry Q No:

10

Hepatobiliary system

Biochemistry Q No:

52

Neurology

Biochemistry Q No:

11

Gastrointestinal system

Biochemistry Q No:

53

Endocrinology

Biochemistry Q No:

12

Hematology

Biochemistry Q No:

54

Hematology

Biochemistry Q No:

13

Dermatology

Biochemistry Q No:

55

Hematology

Biochemistry Q No:

14

Dermatology

Biochemistry Q No:

56

Neurology

Biochemistry Q No:

15

Dermatology

Biochemistry Q No:

57

Hepatobiliary system

Biochemistry Q No:

16

Dermatology

Biochemistry Q No:

58

Hepatobiliary system

Biochemistry Q No:

17

Neurology

Biochemistry Q No:

59

Head and neck

Biochemistry Q No:

18

Endocrinology

Biochemistry Q No:

60

Hematology

Biochemistry Q No:

19

Musculoskeletal

Biochemistry Q No:

61

Hepatobiliary system

Biochemistry Q No:

20

Hepatobiliary system

Biochemistry Q No:

62

Hematology

Biochemistry Q No:

21

Endocrinology

Biochemistry Q No:

63

Head and neck

Biochemistry Q No:

22

Gastrointestinal system

Biochemistry Q No:

64

Musculoskeletal

Biochemistry Q No:

23

Hepatobiliary system

Biochemistry Q No:

65

Hematology

Biochemistry Q No:

24

Renal

Biochemistry Q No:

66

Blood vessels

Biochemistry Q No:

25

Hematology

Biochemistry Q No:

67

Neurology

Biochemistry Q No:

26

Endocrinology

Biochemistry Q No:

68

Neurology

Biochemistry Q No:

27

Musculoskeletal

Biochemistry Q No:

69

Gastrointestinal system

Biochemistry Q No:

28

Neurology

Biochemistry Q No:

70

Musculoskeletal

Biochemistry Q No:

29

Musculoskeletal

Biochemistry Q No:

71

Endocrinology

Biochemistry Q No:

30

Endocrinology

Biochemistry Q No:

72

Endocrinology

Biochemistry Q No:

31

Neurology

Biochemistry Q No:

73

Endocrinology

Biochemistry Q No:

32

Pulmonology

Biochemistry Q No:

74

Hematology

Biochemistry Q No:

33

Hepatobiliary system

Biochemistry Q No:

75

Hematology

Biochemistry Q No:

34

Hematology

Biochemistry Q No:

76

Neurology

Biochemistry Q No:

35

Musculoskeletal

Biochemistry Q No:

77

Endocrinology

Biochemistry Q No:

36

Hematology

Biochemistry Q No:

78

Dermatology

Biochemistry Q No:

37

Endocrinology

Biochemistry Q No:

79

Endocrinology

Biochemistry Q No:

38

Hematology

Biochemistry Q No:

80

Neurology

Biochemistry Q No:

39

Hematology

Biochemistry Q No:

81

Hematology

Biochemistry Q No:

40

Endocrinology

Biochemistry Q No:

82

Endocrinology

Biochemistry Q No:

41

Hematology

Biochemistry Q No:

83

Gastrointestinal system

Biochemistry Q No:

42

Oncology

Biochemistry Q No:

84

Hematology

Biochemistry Q No:

85

Neurology

Biochemistry Q No:

110

Blood vessels

Biochemistry Q No:

86

Neurology

Biochemistry Q No:

111

Hematology

185

USMLE WORLD STEP 1

BIOCHEMISTRY

Biochemistry Q No:

87

Pulmonology

Biochemistry Q No:

112

Head and neck

Biochemistry Q No:

88

Musculoskeletal

Biochemistry Q No:

113

Endocrinology

Biochemistry Q No:

89

Neurology

Biochemistry Q No:

114

Musculoskeletal

Biochemistry Q No:

90

Hematology

Biochemistry Q No:

115

Neurology

Biochemistry Q No:

91

Neurology

Biochemistry Q No:

116

Gastrointestinal system

Biochemistry Q No:

92

Hepatobiliary system

Biochemistry Q No:

117

Hematology

Biochemistry Q No:

93

Hematology

Biochemistry Q No:

118

Endocrinology

Biochemistry Q No:

94

Hepatobiliary system

Biochemistry Q No:

119

Gastrointestinal system

Biochemistry Q No:

95

Neurology

Biochemistry Q No:

120

Gastrointestinal system

Biochemistry Q No:

96

Neurology

Biochemistry Q No:

121

Neurology

Biochemistry Q No:

97

Musculoskeletal

Biochemistry Q No:

122

Hematology

Biochemistry Q No:

98

Cardiology

Biochemistry Q No:

123

Hepatobiliary system

Biochemistry Q No:

99

Endocrinology

Biochemistry Q No:

124

Hematology

Biochemistry Q No:

100

Cardiology

Biochemistry Q No:

125

Gastrointestinal system

Biochemistry Q No:

101

Hematology

Biochemistry Q No:

126

Dermatology

Biochemistry Q No:

102

Oncology

Biochemistry Q No:

127

Oncology

Biochemistry Q No:

103

Musculoskeletal

Biochemistry Q No:

128

Dermatology

Biochemistry Q No:

104

Neurology

Biochemistry Q No:

129

Endocrinology

Biochemistry Q No:

105

Endocrinology

Biochemistry Q No:

130

Hematology

Biochemistry Q No:

106

Endocrinology

Biochemistry Q No:

131

Neurology

Biochemistry Q No:

107

Hematology

Biochemistry Q No:

132

Hematology

Biochemistry Q No:

108

Neurology

Biochemistry Q No:

133

Hepatobiliary system

Biochemistry Q No:

109

Hematology

Biochemistry Q No:

134

Hematology

186

USMLE WORLD STEP 1

BIOCHEMISTRY

Q NO 1: A 34-year-old female develops moderate hyperglycemia during her first


pregnancy and is diagnosed with gestational diabetes mellitus. Her glycemic
status improves markedly after delivery. She says that her mother and younger
sister both had high blood sugars during pregnancy. If this patients gestational
hyperglycemia is genetically predisposed, this patient is most likely to have
decreased activity in which of the following enzymes?
A. Glucokinase
B. Phosphofructokinase
C. Aldolase
D. Pyruvate kinase
E. Enolase
F. Pyruvate carboxylase
G. Lactate dehydrogenase
Explanation:
Glucose is the major stimulant of insulin secretion. After glucose enters the beta
cells, it is metabolized by glycolysis, followed by citric acid cycle, which results in the
generation of ATP molecules. A high ATP to ADP ratios within the beta cells results in
the closure of potassium (KATP) channel, which causes insulin secretion by opening
the voltage- dependent calcium channel.
In the beta cell of the pancreas, the first enzyme in the glycolytic pathway is
glucokinase, instead of hexokinase the first enzyme of other tissues. Glucokinase
converts glucose to glucose 6ophosphate. Glucokinase has a much higher Km (10
mmol) than hexokinase (0.5 mmol). Further more, glucokinase is less sensitive to
allosteric inhibition by its product, glucose 6-phosphate. Glucokinase serves as a
major glucose-sensor for the beta cell. An increase in serum glucose leads to
increased metabolism of glucose within the beta cells ultimately leading to insulin
secretion by the formation of ATP. The conversion of glucose to glucose-6- phosphate
by glucokinase is crucial and rate limiting. Once metabolized to glucose 6-phosphate,
glucose is rapidly metabolized in the glycolytic and citric cycle.
Glucokinase defects result in one of the types of maturity-onset diabetes.
Inactivating mutations of the glucokinase gene lead to a decrease in the enzymes
function, causing decrease in the metabolism of glucose, lesser ATP formation, and
diminished insulin secretion.
(Choices A - G) The other choices listed are enzymes involved in glycolytic pathways.
Diabetes mellitus is not a complication of defects in these enzymes.
Educational Objective:
Glucokinase is a glucose sensor within pancreatic beta cells. Inactivating mutations
of the enzyme result in mild hyperglycemia that can be exacerbated by pregnancy.

187

USMLE WORLD STEP 1

BIOCHEMISTRY

Q NO 2: A newborn is diagnosed with hyperphenylalaninemia at birth and placed on


a special phenylalanine-restricted diet. Several months later, the infant has a normal
serum phenylalanine level; however, some neurological abnormalities are observed.
His serum prolactin level is elevated. Which of the following enzymes is most likely
deficient in this patient?
A. Phenylalanine hydroxylase
B. Dihydrobiopterin reductase
C. Dopamine hydroxylase
D. Homogentisate oxidase
E. N-methyltransferase
F. Tyrosinase

Explanation:
Deficiency of dihydrobiopterin reductase, the enzyme responsible for reduction of
dihydrobiopterin (BH2) to BH4, is the most common cause for a deficiency of BH4.
Tetrahydrobiopterin (BH4) is a cofactor for enzymes that participate in the synthesis
of tyrosine, DOPAI and serotonin, as well as nitric oxide. Under normal conditions
dopamine from the tuberoinfundibular system typically inhibits the release of
prolactin. Decreased BH4 causes decreased levels of dopamine, which therefore
cause increased levels of prolactin.

188

USMLE WORLD STEP 1

BIOCHEMISTRY

The above image illustrates the biochemical reaction whereby dopamine is


synthesized from tyrosine. The initial step in this pathway is the conversion of
tyrosine to DQPAI catalyzed by the enzyme tyrosine hydroxylase, which uses BH4 as
a cofactor. Subsequently DOPA is decarboxylation to dopamine by the enzyme DOPA
carboxylase. Dopamine ultimately serves as the precursor molecule to the
catecholamines epinephrine and norepinephrine.
(Choice A) Phenylalanine hydroxylase is an enzyme that converts ingested
phenylalanine to tyrosine. The above patients phenylalanine levels have normalized
with diet therapy but he has low dopamine levels. Patients with classic
phenylketonuria, or phenylalanine hydroxylase deficiency, do not have any metabolic
error when producing dopamine; therefore, this patient must have dihydrobiopterin
reductase deficiency, whicheven with phenylalanine restriction causes low
dopamine levels (see diagram).
(Choice C) Dopamine hydroxylase is the enzyme that catalyzes the biosynthesis of
norepinephrine from dopamine. It does not use BH4 is a cofactor.
(Choice D) Alkaptonuria is an autosomal-recessive disorder caused by a deficiency of
the enzyme homogentisic acid oxidase, which normally breaks down homogentisic
acid (also called alkapton), a toxic tyrosine by product that is harmful to bones and
cartilage.
(Choice E) N-methyltransferase converts norepinephrine into epinephrine.
(Choice F) Albinism is caused by defects in the biosynthesis and distribution of
melanin. Melanin is synthesized in melanocytes from tyrosine by the enzyme
tyrosinase.
Educational Objective:
Tetrahydrobiopterin (BH4) is a cofactor used in the synthesis of tyrosine, dopa,
serotonin, and nitric oxide. Initially, tyrosine is converted to DOPA by the enzyme
tyrosine hydroxylase, with BH4 present as a cofactor. Next, DOPA is decarboxylated
to dopamine by the enzyme DOPA carboxylase.

189

USMLE WORLD STEP 1

BIOCHEMISTRY

Q NO 3: Oxaloacetate forms aspartate while reacting with glutamate. Which of the


following most likely participates in this reaction?
A. Thiamine
B. Riboflavin
C. Niacin
D. Pyridoxine
E. Biotin
F. Folic acid

Explanation:
Pyridoxine (vitamin B6) is an essential cofactor for the transamination and
decarboxylation of amino acids for gluconeogenesis, and for other essential
biochemical processes. Transamination reactions typically occur between an amino
acid and an -keto acid. The amino group is transferred to the a-keto acid from the
amino acid, and the -keto acid thereby becomes an amino acid.
Thus, in this question oxaloacetate (a-keto acid) reacts with glutamate (amino acid)
to form aspartate (the resulting amino acid) and -ketoglutarate (the resulting aketo acid). Transaminases (amino transferases) are the enzymes that catalyze

190

USMLE WORLD STEP 1

BIOCHEMISTRY

transamination reactions, and pyridoxal phosphate (vitamin B6) serves as an


essential cofactor for the transaminase.
(Choice A) Thiamine (vitamin B1) serves as a coenzyme for a number of important
enzymes including transketolase, a-ketoglutarate dehydrogenase, and pyruvate
dehydrogenase.
(Choice B) Riboflavin (vitamin B2) is used in dehydrogenase reactions involving FMN
and FAD.
(Choice C) Many dehydrogenases use NAD and NADP, which are formed from niacin.
Niacin (vitamin B3, or nicotinic acid) deficiency is known as pellagra and is classically
associated with the four Ds: dermatitis, dementia, diarrhea, and, if untreated, death.
(Choice E) Biotin is a water-soluble vitamin that is a cofactor for all four carboxylase
enzymes: pyruvate carboxylase, acetyl CoA carboxylase, propionyl CoA carboxylase,
and 3-methylcrotonyl-CoA-carboxylase.
(Choice F) Folic acid is an essential cofactor in nucleic acid synthesis, and a
deficiency of either folate or vitamin B12 results in megaloblastic anemia.
Educational Objective:
Transamination reactions typically occur between an amino acid and an -keto acid.
The amino group from the amino acid is transferred to the a-keto acid, and the aketo acid in turn becomes an amino acid. Pyridoxal phosphate (vitamin B6) serves
as a cofactor in amino acid transamination and in decarboxylation reactions.

191

USMLE WORLD STEP 1

BIOCHEMISTRY

Q NO 4: An agent that specifically blocks the interaction of inositol triphosphate with


its intracellular receptor would most likely decrease the activity of:
A. Phospholipase C
B. Lipoxygenase
C. Protein kinase C
D. Phosphodiesterase
E. Adenylate cyclase

Explanation:
The G-protein-coupled-receptors have a very characteristic structure with seven
transmembrane regions, an extracellular domain and an intracellular domain coupled
with the trimeric G-protein. In their inactivated state G-proteins exist as
heterotrimers consisting of alpha, beta and gamma subunits with guanosine
diphosphate (GDP) tightly bound to the alpha subunit. G proteins are activated after
binding of hormone to the extracellular domain. The first step in activation of a Gprotein occurs when GDP is exchanged for GTP on the alpha subunit. Once bound to
GTP, the alpha subunit dissociates from the beta and gamma subunits and exposes
its catalytic domain for either adenylate cyclase or phospholipase C depending on
the ligand.
If the S-protein alpha subunit activates phospholipase C, then the degradation of
phosphatidylinositol 4, 5- bisphosphate to inositol 1, 4, 5-triphosphate (IP3) and
diacylglycerol (DAG) occurs. Diacylglycerol stimulates protein kinase C, which is
responsible for some intracellular effects. Inositol 1, 4, 5-triphosphate (IP3)
produces most of the intracellular effects of this pathway by increasing intracellular
calcium, and elevated intracellular calcium activates protein kinase C. If the action of

192

USMLE WORLD STEP 1

BIOCHEMISTRY

IP3 were blocked as described in the question stem, then decreased activation of
protein kinase C would occur upon hormone binding. (Choice C).
(Choice A) The activity of phospholipase C would be unchanged if IP3 were blocked
because phospholipase C exerts its effect before IP3 in the calcium /
phosphatidylinositol second messenger system.
(Choice B) Lipoxygenase is an enzyme responsible for formation of leukotrienes from
arachidonic acid. It is not involved in intracellular signaling.
(Choice D) Termination of the effects of hormones that act by cAMP or cGMP Gprotein second messenger systems is carried out by the enzyme phosphodiesterase.
Phosphodiesterase has no effect on the IP3 second messenger system.
(Choice E) Activation of adenylate cyclase leads to the formation of cyclic AMP and
the subsequent activation of protein kinase A. Protein kinase A activates the proteins
that produce the intracellular effects of hormones.
Educational Objective:
After a hormone binds a G-protein coupled receptor that activates phospholipase C,
the initial step of the IP3 second messenger system involves degradation of
membrane lipids into diacylglycerol (DAG) and inositol triphosphate (IP3) by that
enzyme. Protein kinase C is activated by DAG as well as calcium released from
sarcoplasmic reticulum under the influence of 1P3.

193

USMLE WORLD STEP 1

BIOCHEMISTRY

Q NO 5: An infant suffering from severe neurological symptoms and lactic acidosis is


diagnosed with pyruvate dehydrogenase complex deficiency. When placed on a
special diet, the patient demonstrates some decrease in the blood lactate level.
Which of the following substances can be most safely supplemented to this patient?
A. Glycerol
B. Alanine
C. Galactose
D. Lysine
E. Asparagine
F. Serine
Explanation:
Pyruvate dehydrogenase (PDH) is an allosteric enzyme that converts pyruvate into
acetyl-CoA in the presence of oxygen (i.e. during aerobic metabolism). In the
absence of oxygen or with a deficiency of PDHI pyruvate is alternatively converted to
lactate by the enzyme lactate dehydrogenase. Excessive lactate production in these
states results in lactic acidosis.
PDH deficiency has a wide spectrum of presentations. Because carbohydrates may
aggravate lactic acidosis, a ketogenic diet is recommended in these patients. Amino
acid catabolism following removal of the amino group results in formation of
intermediates that are either glucogenic (producing intermediates of the citric acid
cycle or pyruvate) or ketogenic (producing acetoacetate or its precursors). Some
amino acids such as phenylalanine, isoleucine and tryptophan are both glucogenic as
well as ketogenic. Leucine and lysine (Choice D) are exclusively ketogenic and would
not lead to increased formation of lactic acid. Lysine is an essential amino acid that
is totally ketogenic.
(Choice A) Glycerol forms the backbone to triglycerides, and in the fasting state
when triglycerides are degraded for energy, glycerol is released to be used in
gluconeogenesis. Glycerol can aggravate lactic acidosis in patients with pyruvate
dehydrogenase deficiency because it can be converted to dihydroxyacetone
phosphate, which in turn can form pyruvate and subsequently lactate.
(Choices B and F) Alanine is a non-essential glucogenic amino acid that can be
transaminated to form pyruvate. Similarly, serine is a non-essential amino acid that
can be converted to pyruvate by serine hydratase.
(Choice C) Galactose, when combined with glucose as a disaccharide, forms the milk
sugar lactose. Galactose is metabolized in the glycolytic pathway, thus galactose
feeding will result in aggravation of lactic acidosis in patients with pyruvate
dehydrogenase deficiency.
(Choice E) Asparagine is a non-essential glucogenic amino acid that is catabolized
initially to aspartate by the enzyme asparaginase. Aspartate is transaminated with
alpha ketoglutarate to produce glutamate and oxaloacetate, a gluconeogenic
intermediate and an intermediate in the TCA cycle.
Educational Objective: Pyruvate dehydrogenase deficiency is a disease with multiple
possible presentations ranging from neonatal death to mild episodic symptoms in
adulthood. By preventing the conversion of pyruvate to acetyl C0A, pyruvate is
shunted to lactic acid resulting in lactic acidosis in these patients. Lysine and leucine
are exclusively ketogenic and would not increase the blood lactate level in patients
suffering from pyruvate dehydrogenase deficiency.

194

USMLE WORLD STEP 1

BIOCHEMISTRY

Q NO 6: A sample of liver parenchyma is homogenized and centrifuged to remove


membrane components and or gandles leaving only the cytosol and the proteins it
contains. Which of the following enzyme activities is most likely present in the
homogenate?
A. Pyruvate carboxylase
B. Ornithine transcarbamoylase
C. Transketolase
D. Succinate dehydrogenase
E. 3-Hydroxy-3-methylglutaryl-CoA lyase
Biochemical processes for cellular metabolism occur in distinct locations within cells.
For example b-oxidation of fatty acids ketogenesis, the citric acid cycle parts of the
urea cycle (carbamoyl phosphate synthetase and ornithine transcarbamoylase) and
pyruvate carboxylation all occur exclusively with in the mitochondria. Transketolase
is an enzyme of the pentose phosphate pathway that uses thiamine (Vitamin B1) as
a cofactor. All of the reactions of the pentose phosphate pathway occur in the
cytoplasm, so the enzyme transketolase would remain in the homogenate described
in the question stem while the other enzymes mentioned in the other answer choices
would be removed because they all reside within the mitochondria.
(Choice A) Pyruvate carboxylase catalyzes the initial step in gluconeogenesis by
converting pyruvate to oxaloacetate. This enzyme requires biotin as a cofactor and
functions within the mitochondria.
(Choice B) Ornithine transcarbamoylase catalyzes the second step of the urea cycle
where ornithine and carbamoyl phosphate are combined to form citrulline. This
reaction occurs within the mitochondria. Deficiency of this enzyme leads to
hyperammonemia and mental retardation.
(Choice D) Succinate dehydrogenase is an enzyme of the TCA cycle that functions to
convert succinate to fumarate. It is an inner mitochondrial membrane protein and
functions as part of the electron transport chain.
(Choice E) 3-Hydroxy-3-methylglutarql-CoA (HMG C0A) lyase is a mitochondrial
enzyme that is responsible for ketogenesis from HMG CoA. HMG CoA results from
the degradation of leucine, a strictly ketogenic amino acid, and also from synthesis
by HMG CoA synthase. This molecule is also an intermediate in the formation of
cholesterol.
Educational Objective:
Biochemical processes for cellular metabolism occur in distinct locations within cells.
For example b-oxidation of fatly acids ketogenesis, the citric acid cycle, parts of the
urea cycle (carbamoyl phosphate synthetase 1 and ornithine transcarbamoylase)
and pyruvate carboxylation all occur exclusively within the mitochondria. All of the
reactions of the pentose phosphate pathway occur in the cytoplasm.

195

USMLE WORLD STEP 1

BIOCHEMISTRY

Q NO 7: A 7-year-old patient presents to your office accompanied by his parents. He


has been hospitalized multiple times for painful episodes in his hands and feet over
the last several years. He has no known medical problems and takes no medications
except for acetaminophen for pain control. You suspect that he has a valine for
glutamic acid substitution at position 6 of the -globin chain of the hemoglobin
molecule. This patients hemoglobin would most likely aggregate upon:
A. 3-chain folding
B. Interaction with HbF
C. Proton release
D. Oxygen unloading
E. 23-bisphosphoglycerate depletion
Explanation:
Sickle cell anemia (HbS) is the most common inherited blood disorder in the United
States. In HbS, the nonpolar amino acid valine replaces the charged amino acid
glutamate at position 6 of the beta chain. This results in alteration of a hydrophobic
part of the beta chain that fits into a complementary site on the alpha chain of
another hemoglobin molecule. The altered structure results in aggregation of
hemoglobin molecules under anoxic conditions. After polymerization HbS initially
forms a gel followed by a meshwork of fibrous polymers that causes the red blood
cells to distort into an abnormal sickle shape. Sickling is promoted by conditions
associated with low oxygen levels, increased acidity, or low blood volume
(dehydration).
Organs in which blood moves slowly (e.g. spleen, liver, kidney) are predisposed to
lower oxygen levels or acidity. In addition, organs with particularly high metabolic
demands (e.g brain, muscles, and placenta) promote sickling by extracting more
oxygen from the blood. These conditions make these organs susceptible to injury
from sickle cell anemia. Sickled cells are not flexible enough to pass through
microvasculature. By impeding blood flow through these vascular beds, the sickled
cells cause microinfarcts in tissues and painful vasoocclusive crises in the bone.
(Choice A) Globin chains of the hemoglobin tetramer are compactly folded with
nonpolar hydrophobic residues in the interior and charged polar residues on the
surface. Each chain has several alpha helical stretches (secondary structures) and
beta bends (tertiary structure). Substitution of valine for glutamic acid does not
result in significant change in beta folding.
(Choice B) Clinical manifestations of sickle cell disease can be less severe in patients
who also produce larger amounts of HbF in adulthood. Such patients include some
people of Middle Eastern descent. In fetal life and early infancy, HbF is the
predominant hemoglobin in patients who produce a-globin. HbF contains two alpha
and two gamma globin chains. HbS does not polymerize when HbF is present, so
patients with sickle cell disease often do not have symptoms until the HbF fraction
decreases a few months following delivery.
(Choice C) Protons are released from deoxyhemoglobin in the lungs where the high
p02 favors release of CO2 and binding of oxygen to hemoglobin. HbS does not sickle
in environments with high oxygen content.
(Choice E) 213-DPG binds the two beta chains by ionic bonding and stabilizes the
taut (T) deoxyhemoglobin. This binding decreases the oxygen affinity of hemoglobin
and facilitates the release of oxygen at the tissue level. With depletion of 2, 3 DPGI
the affinity of hemoglobin for oxygen will increase and result in uptake of oxygen by

196

USMLE WORLD STEP 1

BIOCHEMISTRY

hemoglobin. Oxygenated hemoglobin S does not polymerize; thus, sickling of


erythrocytes will be decreased.
Educational Objective:
Hemoglobin S (HbS) aggregates in the deoxygenated state. HbS polymers form
fibrous strands that reduce red blood cell membrane flexibility and promote sickling.
Sickling occurs under all conditions associated with anoxia including low pH and high
2I3-DPG. These inflexible erythrocytes predispose to microvascular occlusion and
micro infarction.

197

USMLE WORLD STEP 1

BIOCHEMISTRY

Q NO 8: Small circular molecules of DNA are isolated from exocrine pancreatic cells.
These molecules resemble bacterial DNA and code for proteins, transfer RNA, and
ribosomal RNA. The molecules have been isolated from which of the structures
marked on the slide below?

A.
B.
C.
D.
E.

A
B
C
D
E

Explanation:
Most DNA in human cells is found within the nucleus: however mitochondria have
their own DNA called mitochondrial DNA (mtDNA). Human mtDNA has 37 genes
coding for thirteen proteins two forms of rRNA, and 22 forms of tRNA. mtDNA is
involved in the production of some of the proteins involved in oxidative metabolic
pathways while nuclear DNA encodes for most of the proteins actually found within
mitochondria. All mitochondria, and therefore all mtDNA, are maternally derived
from the oocyte and under goes minimal changes during transfer from mother to
offspring. Thus, the genetic code of mtDNA is slightly different from nuclear DNA.
For example UGA signifies a stop codon in nuclear DNA but codes for tryptophan in
mtDNA. The existence of mitochondrial DNA that is separate from nuclear DNA
supports the theory that eukaryotic cells first appeared evolutionary when a
prokaryotic cell was absorbed into another cell without being digested forming a
symbiotic relationship. This theory is known as the endosymbiotic theory. Because of
this unique genetic transfer of mitochondrial DNA diseases arising from mutations in
mitochondrial DNA are transmitted from the mother to all of her offspring.
(Choice A) The folded membranous system identified here has a rough. stippled
appearance secondary to the presence of numerous ribosomes bound to its
membranes which are involved in the synthesis of proteins destined for export or
packaging into granules or organelles.
(Choice B) The dark region identified within the nucleus is the nucleolus the site of
synthesis and assembly of ribosomal components. Remember that the nucleolus is
not membrane bound.
(Choice C) The lighter electron lucent regions within the nucleus identify
heterochromatin, or DNA that has been un packaged and is actively being
transcribed.
(Choice E) This electron dense membrane bound spherical structure represents an
exocrine secretory granule containing exocrine enzymes and other proteins
packaged for secretion.
Educational Objective:
Resembling prokaryotic DNA and being derived completely from the mother
mitochondrial DNA (mtDNA) is the most common non-nuclear DNA found in
eukaryotic cells.

198

USMLE WORLD STEP 1

BIOCHEMISTRY

Q NO 9: A 44-year-old homeless male develops confusion and ophthalmoplegia.


Upon review of the medical record, you discover that he has been admitted to the
hospital with alcohol intoxication several times before. Which of the following
reactions is most likely impaired in this patient?

A.
B.
C.
D.
E.
F.
G.

A
B
C
D
E
F
G

199

USMLE WORLD STEP 1

BIOCHEMISTRY

Explanation:
Thiamine serves as a cofactor for a number of important enzymes including
transketolase, alpha-ketoglutarate dehydrogenase, and pyruvate dehydrogenase.
Patients with alcoholism are classically deficient in thiamine and administration of
glucose to these patients without first administering a thiamine supplement results
in the development of the acute neurological complications of thiamine deficiency
known as Wernicke encephalopathy. Clinically Wernicke encephalopathy is
characterized by acute confusion ophthalmoplegia and ataxia. Treatment with
thiamine usually results in prompt resolution of symptoms.
The bulk of ATP used for cellular processes is generated by oxidation of pyruvate
through the tricarboxylic acid cycle (TCA cycle). The enzymes of the TCA cycle are
located in the mitochondrion. Pyruvate generated from glycolysis is converted to
acetyl C0A by the enzyme pyruvate dehydrogenase, and acetyl CoA then enters the
TCA cycle.
The first reaction of the TCA cycle is the condensation of acetyl-CoA with
oxaloacetate to form citrate (choice G) catalyzed by the enzyme citrate synthase.
Citrate is then converted to isocitrate by the enzyme aconitase, an iron sulfurcontaining enzyme. Isocitrate dehydrogenase then decarboxylates isocitrate to
alpha-ketoglutarate. Isocitrate dehydrogenase is a rate limiting step in the TCA cycle
and it is also the step where the first NADH is produced. This enzyme is negatively
regulated by NADH and ATP and positively regulated by isocitrate, ADP and AMP.
The conversion of alpha-ketoglutarate to succinyl-CoA involves oxidative
decarboxylation by the enzyme alpha ketoglutarate dehydrogenase (choice B). This
reaction generates a second NADH. Alpha-ketoglutarate dehydrogenase is an
enzyme complex requiring several cofactors, including thiamine, for its action.
The conversion of succinyl-CoA to succinate by succinyl-CoA synthetase (choice C)
involves use of the high-energy thioester present in succinyl-CoA to drive nucleotide
triphosphate synthesis by a process known as substrate-level phosphorylation. In
this process, high-energy phosphate intermediates are formed that eventually
phosphorylate GDP to form GTP.
Succinyl dehydrogenase (choice D) catalyzes the conversion of succinate to fumarate
with sequential reduction of enzyme bound flavin adenine dinucleotide (FAD).
Fumarate hydratase (fumarase) catalyzes the formation of malate from fumarate
(choice E). Malate is subsequently converted to oxaloacetate by malate
dehydrogenase, and this is the final step of the TCA cycle (choice F).
Educational Objective:
-Ketoglutarate dehydrogenase requires thiamine as a cofactor. Administration of
glucose to thiamine deficient patients, such as alcoholics, will result in Wernicke
encephalopathy due to increased thiamine demand.

200

USMLE WORLD STEP 1

BIOCHEMISTRY

Q NO 19: An infant suffering from lethargy and vomiting is diagnosed with a


deficiency of several mitochondrial enzymes. A common subunit containing
lipoic acid is abnormal in these enzymes. The patient is most likely to have
which of the following findings?

A.

Citrullinemia
B. Lactic acidosis
C. Methylmalonic aciduria
D. Homocystinuria
E. Orotic aciduria

Explanation:

201

USMLE WORLD STEP 1

BIOCHEMISTRY

Coenzymes and cofactors are relatively small molecules required for many enzymes
to perform their catalytic functions. Lipoic acid (LA) is involved in the
decarboxylation of alpha ketoacid and the transfer of alkyl groups. The transfer of an
alkyl group from pyruvate to coenzyme A is essential for the function of pyruvate
dehydrogenase (PDH).
PDH requires five coenzymes: CoA, FAD, lipoic acid NAD and thiamine
pyrophosphate (TPP). In the absence of these coenzymes, PDH is unable to carry out
its function of converting pyruvate to acetyl CoA for metabolism in the TCA cycle.
Decreased functioning of pyruvate dehydrogenase will increase the conversion of
pyruvate to lactate by the enzyme lactate dehydrogenase in an effort to regenerate
NADI and this will eventually lead to lactic acidosis. Lipoic acid also acts as a
coenzyme for alpha ketoglutarate dehydrogenase and branched-chain ketoacid
dehydrogenase.
(Choice A) Citrullinemia is a urea cycle disorder that results from deficiency of
argininosuccinate synthetase. ATP is the cofactor required by argininosuccinate
synthetase.
(Choice C) Methylmalonic aciduria or acidemia results in acidemia due to increased
circulating levels of methylmalonic acid (organic acidemia). This generally results
from a deficiency of the vitamin B12-dependent enzyme methyl malonate mutase.
(Choice D) Homocystinuria results from a deficiency of cystathionine synthase and is
characterized by premature atherosclerosis. Homocysteine is disposed of by the
body by two pathways: 1. conversion to cysteine by the combined actions of two
vitamin B6-requiring enzymes cystathionine synthase and cystathionase, and 2.
conversion to methionine by a folate and vitamin B12-dependent process.
(Choice E) Orotic aciduria occurs due to defective pyrimidine synthesis resulting from
a deficiency of the enzyme orotate phosphoribosyl transferase. This enzyme requires
glutathione as a coenzyme.
Educational Objective:
Lipoic acid is a cofactor for several mitochondrial enzymes: PDH (deficiency results
in lactic acidosis), a-ketoglutarate DH and branched-chain ketoacid DH (deficiency
results in maple syrup urine disease).

202

USMLE WORLD STEP 1

BIOCHEMISTRY

Q NO 11: A 6-year-old male who lives in an old house suffers from irritability,
constipation and anemia. His high blood level of -aminolevulinic acid is
maintained secondary to the presence of which of the following cofactors?
Thiamine pyrophosphate
B. Pyridoxal phosphate
C. Lipoic acid
D. Retinoic acid
E. Biotin

A.

Explanation:
Lead is a soft heavy metal used in the manufacture of a wide range of items,
including batteries alloys, and ammunition. The most significant means of
occupational exposure is inhalation. Consumers can also be exposed to lead through
contact with dust in lead-painted homes (old homes), lead-contaminated soil water
that traverses lead plumbing, lead-glazed pottery, and homemade alcoholic
beverages (moonshine). The most significant means of exposure for the consumer is
gastrointestinal absorption.
Lead overdose causes toxicity through four distinct mechanisms. Perhaps most
importantly, lead has a strong affinity for sulfhydryl groups. This affinity results in
the inhibition of enzymes that incorporate iron into the heme molecule (5aminolevulinic acid dehydratase and ferroketolase). -aminolevulinic acid is formed
when succinyl-C0A and glycine combine in the presence of the cofactor pyridoxal
phosphate. The blood and urinary levels of this enzyme are increased in individuals
with lead poisoning.
(Choice A) Thiamine pyrophosphate is an essential vitamin Bi-derived cofactor that
plays a role in multiple pathways of energy metabolism (eg, pyruvate and aketoglutarate dehydrogenase catalyzed reactions).
(Choice C) Lipoic acid, or lipoate, is an essential cofactor in numerous facets of
aerobic metabolism (eg, transfer of acyl and methylamine groups in 2-oxoacid
dehydrogenase and glycine cleavage complexes, respectively). It is also an
antioxidant capable of scavenging reactive oxygen species and reducing metabolites.
(Choice D) Retinoic acid binds to receptor proteins, and the subsequent retinoic acidprotein complex interacts with and affects expression of numerous genes that affect
growth and differentiation.
(Choice E) Biotin is the cofactor necessary for many reactions involving carboxylation
enzymes (eg, acetyl-CoA carboxylase and pyruvate carboxylase).
Educational Objective:
Pyridoxal phosphate is a necessary cofactor in the synthesis of -aminolevulinic acid
(which is elevated in cases of lead poisoning).

203

USMLE WORLD STEP 1

BIOCHEMISTRY

Q NO 12: A 21-year-old laboratory worker experiences rapid-onset breathing


difficulty palpitations, and flushed skin. He has no significant past medical
history and takes only loratadine for seasonal allergies. The patient is
suspected to have accidental poisoning. Amyl nitrite from a laboratory safety
kit is immediately administered via inhalation. Amyl nitrite affects the affinity
of hemoglobin for which of the following?
Carbon dioxide
B. 2, 3-biphosphoglycerate
C. Carbon monoxide
D. Cyanide
E. lorn
E. Lead

A.

Explanation:
Cyanide binds to a variety of iron-containing enzymes, the most important of which
is the cytochrome a-a3 complex. This complex is critical for electron transport during
oxidative phosphorylation. By binding to this molecule, minute amounts of cyanide
can inhibit aerobic metabolism and rapidly result in death.
The typical clinical syndrome present in cyanide poisoning is rapidly-developing
cutaneous flushing, tachypnea, headache, and tachycardia, often accompanied by
nausea/vomiting, confusion, and weakness. Respiratory distress and cardiac
dysfunction may follow. Laboratory studies indicate severe lactic acidosis in
conjunction with a lessened difference between arterial and venous content (i.e. the
venous blood is still highly oxygenated).

The antidotal effect of nitrites for cyanide poisoning has been recognized since the
late nineteenth century. Nitrites are oxidizers, and act primarily in cyanide poisoning
by inducing the formation of methemoglobin. This occurs when ferrous iron in
hemoglobin is oxidized to ferric iron. Methemoglobin cannot carry oxygen, but it
does have a high affinity for cyanide. Methemoglobin can bind and sequester cyanide
in the blood thereby keeping the poison away from mitochondrial (and other)
enzymes where cyanide exerts its toxic effects. Sodium thiosulfate is also used for
cyanide poisoning: it combines with cyanide to form the less-toxic thiocyanate,
which is excreted in the urine.
(Choices A, B and C) Methemoglobin, which is formed when amyl nitrite is
administered, does not have a high affinity for molecules such as carbon monoxide,
carbon dioxide, and 2,3 bisphosphoglycerate.
(Choice E) The affinity of hemoglobin for iron is not affected by nitrite
administration, though nitrites do oxidize the heme iron to its Fe (III) state.

204

USMLE WORLD STEP 1

BIOCHEMISTRY

(Choice F) Lead poisoning causes defective heme synthesis. Lead poisoning is


treated first and foremost by the avoidance of lead ingestion. If acutely ingested,
chelation therapy, such as with dimercaprol or CaNaEDTA, should be initiated.
Educational Objective:
Nitrites are oxidizing agents that are effective in the treatment of cyanide poisoning
due to their ability to cause methemoglobinemia. Methemoglobin contains ferric
rather than ferrous iron. Cyanide binds to ferric iron more avidly than to
mitochondrial cytochrome enzymes, which saves these mitochondrial enzymes from
cyanides toxic effect.

205

USMLE WORLD STEP 1

BIOCHEMISTRY

Q NO 13: A 12-year-old male is evaluated for ataxia accompanied by episodic


erythematous and pruritic skin lesions and loose stools. Laboratory evaluation
reveals loss of neutral aromatic amino acids in the urine. This patients
symptoms would most likely respond to which of the following supplements?
Thiamine
B. Riboflavin
C. Folic acid
D. Niacin
E. Pyridoxine
F. Tocopherol
G. Ascorbate

A.

Explanation:
At least three specific small bowel enterocyte apical transport proteins appear to be
involved in the absorption of amino acids from the diet. In Hartnup disease, the
intestinal and renal absorption of tryptophan is defective. Tryptophan is an essential
amino acid and a precursor for nicotinic acid serotonin, and melatonin. The clinical
manifestations of Hartnup disease are primarily due to the malabsorption of
tryptophan, resulting in niacin (Vitamin B3) deficiency, because niacin is synthesized
from tryptophan.
Most children with Hartnup disease are asymptomatic, but some children experience
photosensitivity and pellagra-like skin rashes as in the case described above.
Neurologic involvement can occur most commonly leading to ataxia. Neurologic and
skin symptoms typically was and wane during the course of this disease. The main
laboratory finding in Hartnup disease is aminoaciduria, restricted to the neutral
amino acids (alanine, serine, threonine, valine, leucine, isoleucine, phenylalanine,
tyrosine, tryptophan, and histidine). The urinary excretion of proline, hydroxyproline,
and arginine remains unchanged, and this important finding differentiates Hartnup
disease from other causes of generalized aminoaciduria such as Fanconi syndrome.
Treatment with nicotinic acid or nicotinamide and a high- protein diet generally
results in significant improvement of symptoms.
(Choice A) Thiamine use by the body is maximal in states of accelerated
carbohydrate metabolism because it acts as a cofactor for the enzyme transketolase
in the pentose phosphate pathway as well as the enzymes a-ketoglutarate
dehydrogenase and pyruvate dehydrogenase, both of which require thiamine as a
cofactor.
(Choice B) The coenzymes flavin mononucleotide (FMN) and flavin adenine
dinucleotide (FAD) form the prosthetic groups of several enzymes important in
electron transport. Both flavin mononucleotide (FMN) and flavin adenine dinucleotide
(FAD) are synthesized from riboflavin (Vitamin B2). Clinical features of riboflavin
deficiency include cheilosis (perlche), glossitis, keratitis, conjunctivitis,
photophobia, lacrimation, marked corneal vascularization, and seborrheic dermatitis.
(Choice C) Folic acid deficiency may result in megaloblastic anemia, but does not
result in the neurological manifestations of subacute combined degeneration of the
posterior and lateral columns seen specifically with vitamin B12 deficiency.
(Choice E) Pyridoxine (Vitamin B) is converted to pyridoxal-5-phosphate, which acts
as a coenzyme in the decarboxylation and transamination of amino acids. Deficiency
of pyridoxine leads to anemia, peripheral neuropathy, and dermatitis.
(Choice F) Tocopherol (Vitamin E) is a fat-soluble vitamin that functions as a
scavenger of free radicals (antioxidant). Deficiency of Vitamin E is very uncommon,
but when it occurs, it can result in myelopathy or neurologic dysfunction.

206

USMLE WORLD STEP 1

BIOCHEMISTRY

Educational Objective:
Hartnup disease can result in niacin deficiency due to an excess loss of dietary
tryptophan, resulting from defective intestinal and renal tubular absorption of that
amino acid. Remember that niacin (nicotinamide / Vitamin B3) is synthesized from
tryptophan and that tryptophan is an essential amino acid.

207

USMLE WORLD STEP 1

BIOCHEMISTRY

Q NO 14: A 67-year-old Caucasian male who has been drinking a lot recently is
hospitalized with leg edema, skin rash and prominent cognitive changes. He is
diagnosed with multiple vitamin deficiencies. Which of the following is a
precursor of NAD coenzyme?
Cholesterol
B. Carotene
C. Tryptophan
D. Phenylalanine
E. Arginine
E. Orotic acid

A.

Explanation:
Pellagra (which means rough skin in Italian vernacular) is a clinical syndrome
arising secondary to niacin deficiency that is characterized by the three Ds:
dermatitis, diarrhea, and dementia. The dermatitis is usually bilateral and symmetric
on the sun-exposed areas of the body, consisting of roughened thickened, and scaly
skin. The diarrhea arises as a result of columnar epithelium atrophy (and
occasionally ulceration) of the gastrointestinal tract. The dementia develops
secondary to neuronal degeneration in the brain and spinal cord with lesions similar
in appearance to those associated with pernicious anemia.
Niacin (nicotinic acid, or vitamin B3) is an essential component of the coenzymes
nicotinamide adenine dinucleotide (NAD) and nicotinamide adenine dinucleotide
phosphate (NADP). As such niacin plays a pivotal role in the intermediary
metabolism of cells serving to accept electrons or donate hydrogen ions. Specifically
NAD functions as a coenzyme for dehydrogenases involved in the metabolism of fats
carbohydrates, and amino acids; NADPI in contrast is crucial in the hexosemonophosphate shunt of glucose metabolism.
Niacin is either obtained through dietary consumption (eg, grains legumes seed oils,
fruits, vegetables, meats) or is synthesized endogenously from tryptophan. Because
the niacin in corn is in a bound, unabsorbable form populations that primarily subsist
on corn are prone to developing pellagra. In developed countries pellagra is primarily
observed in alcoholics and those suffering from chronic debilitating diseases (eg, HIV
infection). Pellagra can also be seen less commonly in those with carcinoid syndrome
those using isoniazid for prolonged periods, and those with Hartnup disease.
(Choice A) Cholesterol is the precursor to all steroid hormones but is not necessary
for the formation of niacin. (Choice B) Carotene is the precursor to Vitamin A, but is
not necessary for the formation of niacin.
(Choice D) Phenylalanine is the precursor to tyrosine, an amino acid that is
necessary for the formation of catecholamines (but not niacin).
(Choice E) Arginine is the precursor of nitric oxide urea ornithine and agmatine. It is
also necessary for the formation of creatine (but not niacin).
(Choice F) Orotic acid is a precursor of pyrimidine, but is not necessary for the
formation of niacin.
Educational Objective:
Niacin (vitamin B3) can be synthesized endogenously from tryptophan. A deficiency
of this vitamin results in pellagra, which is characterized by dermatitis diarrhea and
dementia.

208

USMLE WORLD STEP 1

BIOCHEMISTRY

Q NO 15: A 20-month-old male is brought to ER with high fever, confusion and a


skin rash suggestive of measles. He has a history of recurrent respiratory
infections over the last 6 months. The patients family has recently emigrated
from a rural Russian province. Which of the following forms of vitamin
supplementation should be considered in this patient?
A. Vitamin A
B. Vitamin K
C. Vitamin D
D. Vitamin E
E. Vitamin B12
F. Vitamin B6
Explanation:
Although rare in the United States and other industrialized nations, vitamin A
deficiency is relatively common in Asia, Africa, and South America. Malnourishment
and fat malabsorption (eg, cystic fibrosis cholestatic liver disease) are the most
significant causes of vitamin A deficiency. Clinical manifestations of the condition
include night blindness complete blindness and xerophthalmia. More unusual findings
include Bitots spots (abnormal squamous cell proliferation and keratinization of the
conjunctiva), corneal perforation, keratomalacia, nonspecific dermatologic
abnormalities and humoral and cell-mediated immune system inhibition via damage
done to phagocytes and T cell lymphocytes. Death can result if the condition is
untreated.
Research has demonstrated that vitamin A is of benefit in treating children afflicted
with measles. Specifically vitamin A therapy reduces the time to recovery from
pneumonia and diarrhea as well as the length of hospital stay and risk of death. It
has been postulated that the vitamin A resolves a virally-induced hyporetinemia,
though this has not yet been confirmed. The WHO recommends that vitamin A be
administered to all children with measles in areas with widespread vitamin A
deficiency or in areas with a measles mortality rate in excess of one percent.
(Choices B, C, D, E, and F) Vitamins K, D, E, and B do not appear to be of direct
benefit in the treatment of measles infection.
Educational Objective:
Vitamin A can be of benefit in the treatment of measles infection.

209

USMLE WORLD STEP 1

BIOCHEMISTRY

Q NO 16: A 79-year-old female presents to your office with leg pain and fatigue.
She lives alone and has lithe money to spend on food. The patient has tibial
subperiosteal hematomas and painful gums. Which of the following nutrient
deficiencies is most likely responsible for this patients symptoms?
A. Vitamin B1
B. Vitamin B2
C. Pyridoxine
D. Folio acid
E. Ascorbic acid
F. Vitamin K
G. Zinc
Explanation:
Vitamin C cannot be synthesized endogenously and therefore must be consumed in
the human diet. This is typically not a problem, as ascorbic acid is abundantly found
in fruits and vegetables (while also being present to a lesser extent in milk, liver, and
fish). Deficiencies of vitamin C are therefore rare in developed countries, but
continue to be a concern in those with inconsistent eating patterns including the
elderly, alcoholics, and persons who live alone.
Vitamin C (ascorbic acid) deficiency eventually results in scurvy, a disease
characterized by hemorrhages, subperiosteal hematomas, bleeding into joint spaces
gingival swelling secondary periodontal infection, anemia, hyperkeratotic papular
rashes, impaired wound healing, and weakened immune response to local infections.
Because ascorbic acid accelerates hydroxylation and amidations reactions, it plays a
crucial role in numerous biosynthetic pathways. One of the most important functions
of ascorbic acid is its activation of prolyl and lysyl hydroxylase precursors, both of
which are necessary for the hydroxylation of procollagen. As collagen contains
considerable hydroxyproline, the quantity and quality of the collagen produced is
dramatically impaired by any reduction in available ascorbic acid.
(Choice A) Vitamin B1 (thiamine) deficiency is characterized by beriberi and
Wernicke syndrome.
(Choice B) Vitamin B2 (riboflavin) deficiency is characterized by cheilosis, stomatitis,
glossitis, dermatitis, corneal vascularization, and ariboflavinosis.
(Choice C) Pyridoxine (vitamin B6) deficiency is characterized by cheilosis, glossitis,
dermatitis, and peripheral neuropathy.
(Choice D) Folic acid deficiency is characterized by megaloblastic anemia and neural
tube defects in the fetus.
(Choice F) Vitamin K deficiency is characterized by a bleeding diathesis (but not
painful gums).
(Choice G) Zinc deficiency is characterized by acrodermatitis enteropathica, growth
retardation, and infertility.
Educational Objective:
The symptoms of scurvy are primarily caused by impaired collagen formation, and
include hemorrhages, subperiosteal hematomas, bleeding into joint spaces, gingival
swelling, secondary periodontal infection, anemia, hyperkeratotic papular rashes,
impaired wound healing, and weakened immune response to local infections.

210

USMLE WORLD STEP 1

BIOCHEMISTRY

Q NO 17: Some proteins that participate in bacterial DNA synthesis have specific
exonuclease activity. Which of the following is the best statement about the 3 to
5 exonuclease activity of DNA polymerase Ill?
A. It cuts DNA at specific DNA sequences
B. It nicks the DNA strands that have formed thymi dine dimers
C. It removes an improper base-pair nucleotide during replication
D. It cleaves DNA strands to relax positive supercoils
E. It can remove groups of nucleotides (up to ten) at a time

Explanation:
During the process of cell division, DNA replication occurs secondary to the
coordinated effects of multiple enzymes and proteins. DNA polymerases are the
primary enzymes responsible for DNA replication, but they can not function without
the assistance of other enzymes such as primase, helicase, ligase, and
topoisomerase I and II. In E. coil, there are three primary types of DNA
polymerases: DNA polymerase I, II, and Ill. Primase forms the 3 OH group primer to
initiate replication of daughter strands while helicase promotes unwinding and
dissociation of the parent strands. On the other hand, topoisomerases reduce
positive and negative super coiling in order to relieve the strain produced by DNA
unwinding.
DNA replication requires a high degree of fidelity; therefore, as synthesis of the
daughter strands proceeds, DNA polymerases proof read to ensure that the daughter
DNA is the exact complement of the parent DNA. All three prokaryotic DNA
polymerases have proof reading activity and remove mismatched nucleotides via a 3
to 5 exonuclease activity (Choice C). Only DNA polymerase I has 5 to 3
exonuclease activity which is used to excise and replace RNA primers and damaged
DNA sequences, which are identified by endonucleases (Choice E).

211

USMLE WORLD STEP 1

BIOCHEMISTRY

(Choice A) In contrast to exonucleases, which remove nucleotides from the end of a


DNA molecule, endonucleases cut DNA at very specific DNA sequences within the
molecule. Restriction endonucleases digest DNA into smaller fragments in a
sequence-specific manner.
(Choice B) One of the major methods of DNA damage by ultraviolet light is the
dimerization of adjacent pyrimidine bases to form thymidine dimers. These dimers
are routinely formed after exposure to sunlight, but are usually removed by
protective enzymatic mechanisms.
Educational Objective:
All three prokaryotic DNA polymerases have proof reading activity and remove
mismatched nucleotides via 3 to 5 exonuclease activity. Only DNA polymerase I has
5 to 3 exonuclease activity which is used to excise and replace RNA primers and
damaged DNA sequences.

212

USMLE WORLD STEP 1

BIOCHEMISTRY

Q NO 18: A 30-year-old female is diagnosed with Hashimotos thyroiditis and


subsequently started on thyroid-replacement therapy. Which of the following
is true about thyroid hormones?
They act by activation of tyrosine kinase.
B. They act primarily by activation of adenylyl cyclase.
C. They have a nuclear receptor.
D. They have a cytoplasmic receptor.
E. They act by increasing the activity of phospholipase C.

A.

Explanation:
The actions of thyroid hormones are mediated by thyroid hormone receptors, which
are located actually with in the nucleus. Nuclear receptors control gene expression
by binding to DNA at hormone-responsive elements in the promoter region of target
genes. Other molecules that act through nuclear receptors include retinoids,
peroxisomal proliferating activated receptors, and fatty acids.
(Choice A) Tyrosine-kinase-activating receptors are located on the cells surface and
usually have an extracellular domain for ligand binding, or trans membrane region,
and carboxy-terminal domain containing tyrosine kinase activity. Insulin and various
growth factors, such as epidermal growth factor and transforming growth factorbeta, work through activation of tyrosine kinase.
(Choices B and E) Several peptide hormones (glucagon, PTHI ACTHI and
gonadotropins) act via these S-protein- coupled membrane-bound receptors. These
receptors typically have seven transmembrane regions spanning the plasma
membrane with one amino-terminal domain and one carboxy-terminal domain. The
extracellular amino- terminal domain is responsible for hormone binding; when this
occurs the cytoplasmic carboxy1erminal actually activates S-protein, which in turn
increases second messengers, such as cyclic AMP by activation of adenylyl cyclase
activity. S-protein can also increase phospholipase C activity resulting in formation of
other second-messengers, like inositol 1 .4,5-tn phosphate and diacylglycerol (DAG).
(Choice D) Receptors for several steroid hormones such as glucocorticoids,
mineralocorticoids, androgens, and estrogens are located primarily in the cytoplasm.
Educational Objective:
Thyroid hormones alter gene transcription by binding to receptors situated inside of
the nucleus. Receptors for several steroid hormones such as glucocorticoids,
mineralocorticoids, androgens, and estrogens are usually initially present in
cytoplasm, although they do migrate to the nucleus once activated.

213

USMLE WORLD STEP 1

BIOCHEMISTRY

Q NO 19: The 12-year-old son of a circus contortionist says that his skin scars and
bruises easily. Physical examination reveals distensible skin and hypermobile joints.
You suspect an inherited defect in which of the following molecules?
A. Fibrillin
B. Laminin
C. Fibronectin
D. Collagen
E. Keratin
F. Elastin
Explanation:
The collagen molecule consists of three polypeptide alpha chains held together by
hydrogen bonds to form a rope-like triple helix structure. The variation in amino acid
sequences in the collagen alpha chains gives rise to collagen diversity in different
tissues. For example type collagen is present in skin, bones, tendons, blood vessels
and the cornea. In these tissues, collagen provides tensile strength and, along with
keratin, is responsible for skin strength and elasticity.
Ehlers-Danlos syndrome (EDS) is a group of rare hereditary disorders involving
connective tissues found in skin, tendons, ligaments, and muscles. EDS usually
manifests clinically as over-flexible (hypermobile) joints over-elastic (hyperelastic)
skin, and fragile tissue susceptible to bruising, wounding, and hemarthrosis. EDS
results from heritable disorders of collagen metabolism. There are several different
types of EDS defined by the extent of involvement of skin joints and other tissues.
Common mutations leading to EDS phenotypes include deficiencies of the lysyl
hydroxylase and pro-collagen peptidase enzymes responsible for collagen synthesis.
(Choice A) Fibrillin- is a major component of the micro fibrils that form a sheath
around elastin. Microfibrils are abundantly present in blood vessels and the
suspensory ligaments of the lens. Defects in the fibrillin- gene cause classic
autosomal dominant Marfans syndrome.
(Choice B) Laminins are heterotrimeric glycoproteins that bind to type IV collagen
underlying epithelial cells. They contribute to the organization and function of the
basal lamina (basement membrane).
(Choice C) Fibronectin binds to integrins on the cell surface and on components of
the extracellular matrix such as collagen and fibrin. It functions as an adhesive
protein involved in cellular differentiation, phagocytosis, platelet adhesion and
thrombus formation.
(Choice E) Keratin is the major protein component of hair and nails in humans.
Keratin is composed of large amounts of alanine and glycine that, because of their
small structure, are able to coil tightly and form large numbers of hydrogen bonds.
Additionally, keratin contains a significant amount of the sulfur-containing amino acid
cysteine, and formation of disulfide bonds between cysteine residues gives additional
rigidity and toughness to the protein. The large amount of keratin in epithelial cells
of skin makes the outmost layer of skin waterproof. A defect in keratins 5 and 14
results in epidermolysis bullosa simplex.
(Choice F) Elastin is a fibrous protein in the connective tissue that gets its name
because of its elastic properties. Elastin fibers can be stretched to several times their
original length but will recoil when stretching forces are withdrawn. Elastin is
synthesized from a polypeptide precursor tropoelastin. Elastin gives elastic
properties to skin, blood vessels and lung alveoli.

214

USMLE WORLD STEP 1

BIOCHEMISTRY

Educational Objective:
Ehlers-Danlos syndrome is a heritable connective tissue disease typically associated
with abnormal collagen. EDS usually manifests clinically as over-flexible
(hypermobile) joints over-elastic (hyperelastic) skin, and fragile tissue susceptible to
bruising, wounding, and hemarthrosis.

215

USMLE WORLD STEP 1

BIOCHEMISTRY

Q NO 20: A 3-year-old male suffers from chronic diarrhea, greasy stool and anemia.
The patient is significantly underweight. He has mild edema of the lower extremities
and oral ulcers. Parenteral feeding and vitamin supplementation are started in this
patient. Pantothenic acid is necessary for which of the following conversions in this
patients metabolism?
A. Glucose to pyruvate
B. Glucose to ribose-5-phosphate
C. Alanine to glucose
D. Oxaloacetate to citrate
F. Glutamate to a-ketoglutarate
Explanation:
The biologically active form of pantothenic acid is coenzyme A, an essential cofactor
in numerous acetylation reactions, including those associated with the tricarboxylic
acid (TCA) cycle. Coenzyme A is particularly important in the first step of the TCA
cycle, as it binds with oxaloacetate to form citrate and then succinyl-CoA. Coenzyme
A is also important in the synthesis of vitamin A, vitamin D, cholesterol, steroids
heme A, fatly acids, amino acids, and proteins.
The pantothenic acid is actively transported into the cell and then undergoes ATPdependent phosphorylation, which transform it into coenzyme A.
Deficiency of pantothenic acid is rare, though it has been observed in severely
malnourished people who complain of paresthesias and dysesthesias (burning feet
syndrome) and gastrointestinal distress.
(Choices A, B, C, and E) These reactions do not require coenzyme A.
Educational Objective:
The biologically active form of pantothenic acid is coenzyme A, which binds with
oxaloacetate in the first step of the TCA (Krebs) cycle to form citrate and then
succinyl-CoA.

216

USMLE WORLD STEP 1

BIOCHEMISTRY

Q NO 21: Activity of an enzyme called aldose reductase in the lens of a healthy


individual produces sorbitol that can not exit the cells. Which of the following is
the most likely product of sorbitol oxidation in these cells?
Glucose
B. Fructose
C. Galactose
D. Galactitol
E. Xylulose

A.

Explanation:
Aldose reductase converts glucose into sorbitol. Sorbitol cannot readily cross cell
membranes and is therefore trapped inside the cells within which it is formed. If the
enzyme sorbitol dehydrogenase is also present in the cell, it can convert sorbitol into
fructose. This pathway is especially active in the seminal vesicles, as sperm use
fructose as their primary energy source.
Other tissues, such as the lens, retina Schwann cells and kidneys have a much lower
activity of sorbitol dehydrogenase. When the level of glucose is low the low activity
of this enzyme is sufficient to convert sorbitol into fructose and disallow sorbitol
accumulation within cells. In states of long-standing hyperglycemia such as in
uncontrolled diabetes mellitus, however an excessive amount of sorbitol is produced
and trapped in these cells. The high concentration of sorbitol within these cells
increases the osmotic pressure and facilitates the influx of water. Osmotic cell injury
resulting from high intracellular sorbitol concentrations is one of the mechanisms for
development of complications of diabetes such as neuropathy and retinopathy.
Additionally retention of water in the lens causes lens opacification and cataract
formation.
(Choice A) Glucose is metabolized into sorbitol by aldose reductase. Sorbitol, in turn,
is converted into fructose by sorbitol dehydrogenase. Glucose is formed in cells by
the processes of glycogenolysis and gluconeogenesis.
(Choices C and D) Another function of aldose reductase is conversion of galactose
into galactitol (i.e. this enzyme converts sugars into their corresponding sugar
alcohols). Normally this pathway is insignificant. In galactosemia (galactose 1phosphate uridyl transferase deficiency) an increased amount of galactitol is
produced resulting in cataract formation.
(Choice E) Xylulose is an end product of glucuronic acid metabolism and an
intermediate in the pentose phosphate pathway.
Educational Objective:
Aldose reductase converts glucose into sorbitol, which is further metabolized into
fructose by sorbitol dehydrogenase. This pathway is the most active in the liver and
seminal vesicles.

217

USMLE WORLD STEP 1

BIOCHEMISTRY

Q NO 22: E. coli colonies grown on a lactose-containing medium up-regulate the


production of the enzymes -galactosidase and galactoside permease. Which of
the following best explains the synchronous production of both enzymes in
response to lactose?
A. There are two activator binding sites for one activator protein
B. There are two operators for one repressor protein
C. There are two repressors for one inducer
D. There are two promoters in close proximity to each other
E. There is one mRNA coding for both enzymes

Explanation:

218

USMLE WORLD STEP 1

BIOCHEMISTRY

The lac operon is the sequence of the E. coil genome which is required for the
metabolism of lactose. The lac operon consists of a regulatory gene (I), promoter
region (p), operator region (o), and three structural genes (z, y, and a). The z gene
codes for -galactosidase (-gal) which is primarily responsible for the hydrolysis of
lactose to glucose and galactose. The y gene codes for permease, a transmembrane
enzyme that increases the permeability of the cell to lactose. The gene encodes a galactoside transacetylase, which transfers acetyl groups to -galactosides and is
unnecessary for lactose metabolism by E. coil.
In prokaryotes, one mRNA transcript contains the sequences for many proteins, and
a single mRNA molecule can be translated into multiple proteins or polypeptides. For
instance, all three proteins of the lac operon -galactosidase, permease, and
transacetylase) are synthesized from a single mRNA molecule containing the z, y
and a gene sequences, respectively. Transcription and translation of the genes of the
lac operon is typically synchronous. Remember that a single mRNA molecule which
codes for more than one protein is referred to as a polycistronic mRNA, and while
most prokaryotic mRNA molecules are polycistronic, eukaryotic mRNA is rarely
polycistronic.
(Choices AD) The lac operon, which codes for all three aforementioned proteins, is
regulated by a single operator, a promoter, and a single group of regulatory
elements: an inducer, repressor, and catabolite activator protein. Modulation of the
transcription of this operon through binding of the operator and action of the
repressor or other regulatory elements will change the transcription of all three lacoperon structural genes (z, y, and a). On the other hand, there are no operators,
repressors, or inducers that can desynchronize the transcription of lac-operon
structural genes.
Educational Objective:
Bacterial mRNA can be polycistronic, meaning that one mRNA codes for several
proteins. An example of polycistronic mRNA is the bacterial lac operon, which codes
for the proteins necessary for lactose metabolism by E. coil; the transcription and
translation of these bacterial proteins is regulated by a single promoter, operator,
and set of regulatory elements.

219

USMLE WORLD STEP 1

BIOCHEMISTRY

Q NO 23: The DNA replication process in eukaryotic cells closely mimics that in
prokaryotic cells, but the volume of genetic material to be replicated is typically
much greater in eukaryotic cells. Which of the following ensures fast DNA
replication in eukaryotic cells?
Energy-independent DNA unwinding
B. Multiple origins of replication
C. No RNA primers synthesized during replication
D. Continuous synthesis of the lagging strand
E. No proofreading by DNA polymerase (pol )

Explanation:
The process of DNA replication is similar in eukaryotes and prokaryotes. The key
steps involved in DNA replication are:
1. Unwinding of double stranded DNA (dsDNA) by helicase to produce single
stranded DNA (ssDNA)
2. Formation of a replication fork
3. Formation of an RNA primer by the action of the enzyme primase
4. Synthesis and concurrent proofreading of daughter DNA strands by DNA
polymerases
5. Ligation of Okazaki fragments on lagging strands by ligase and removal and
replacement of RNA primers with DNA by DNA polymerase I
6. Reconstitution of chromatin and ligation of daughter strands
In E. coil, a prokaryote, the three major types of DNA polymerase are DNA
polymerase I, II and III. In eukaryotes there are five major DNA polymerases:

220

A.

USMLE WORLD STEP 1

BIOCHEMISTRY

alpha, beta, gamma, delta and epsilon. Though the eukaryotic genome is much
larger and more complex than the prokaryotic genome, interestingly the size of the
eukaryotic genome is not the source of its complexity. Its complexity results from
the presence of a large number of non-coding DNA regions between coding regions.
Within genes there are introns (Non-coding regions - Think IN between) separating
exons (Coding regions - Think EX pressed). Prokaryotes rarely have introns within
their genes.
In contrast to prokaryotes which typically have a single origin of replication
eukaryotes have multiple origins of replication? With multiple origins of replication,
the genome can be copied much more quickly because multiple regions are being
replicated at once.
(Choices A, C, D and E) The processes of unwinding synthesis of an RNA primer
synthesis of leading and lagging strands and the proofreading activity of DNA
polymerases are similar in prokaryotes and eukaryotes.
Educational Objective:
Multiple origins of replication make eukaryotic DNA synthesis quick and effective
despite the large size of the genome compared to that of prokaryotic organisms.

221

USMLE WORLD STEP 1

BIOCHEMISTRY

Q NO 24: A 64-year-old male hospitalized with severe abdominal pain and


hypotension begins to hyperventilate. Laboratory testing reveals metabolic
acidosis, an increased anion gap and a high plasma lactate level. This
patients findings are best explained by a low activity of:
Pyruvate kinase
B. Lactate dehydrogenase
C. Pyruvate dehydrogenase
D. Pyruvate carboxylase
E. Enolase

A.

Explanation:
The patient described in the vignette has lactic acidosis secondary to hypoperfusion
of his tissues and subsequent metabolism of glucose via anaerobic glycolysis. In the
process of glycolysis, glucose is ultimately converted to pyruvate. Glycolysis takes
place in the cytosol, and oxidative phosphorylation, which requires the presence of
oxygen, occurs in the mitochondria. The fate of pyruvate generated during glycolysis
is dependent on the presence of oxygen. When inadequate amounts of oxygen are
present in the tissues. pyruvate is converted to lactate by the enzyme lactate
dehydrogenase in order to regenerate the NAD from NADH Ht Thus, under anaerobic
conditions increased amounts of lactate are generated because pyruvate is

222

USMLE WORLD STEP 1

BIOCHEMISTRY

preferentially converted to lactate by the enzyme lactate dehydrogenase. Increased


lactate levels result in metabolic acidosis, and patients with metabolic acidosis will
attempt to compensate by causing a respiratory alkalosis. This is accomplished by
hyperventilation and loss of CO2 (an acid when dissolved in the blood). In the
presence of oxygen, pyruvate is preferentially converted to acetyl coenzyme A by
the enzyme pyruvate dehydrogenase (choice C). Acetyl coenzyme A enters the
mitochondria and undergoes oxidative phosphorylation in the citric acid cycle.
(Choice A) The final step of glycolysis is conversion of phosphoenolpyruvate to
pyruvate by the enzyme pyruvate kinase.
(Choice B) Lactate dehydrogenase is a bidirectional enzyme serves to interconvert
pyruvate and lactate. This enzyme converts pyruvate to lactate in anaerobic
conditions, most classically in exercising skeletal muscle. It also plays a key role in
the liver where lactate, usually generated by working skeletal muscles, is taken up
from the blood and converted to pyruvate for gluconeogenesis.
(Choice D) Pyruvate can also be converted to oxaloacetate by the enzyme pyruvate
carboxylase. This enzyme is involved in regeneration of glucose from pyruvate by
the process of gluconeogenesis.
(Choice E) Conversion of 2-phosphoglycerateto phosphoenolpyruvate is
accomplished by the enzyme enolase.
Educational Objective:
Hypoxia-induced lactic acidosis is caused by a low activity of pyruvate
dehydrogenase (oxidative phosphorylation pathway) and a high activity of lactate
dehydrogenase.

223

USMLE WORLD STEP 1

BIOCHEMISTRY

Q NO 25: A single amino acid residue replacement (serine instead of histidine) in


the hemoglobin -subunit results in poor ionic interaction with 2.3biphosphoglycerate. The resultant hemoglobin has the properties of which of the
following?
A. HbA1C
B. HbS
C. HbC
D. HbF
E. HbH

Explanation:
2, 3-DPG is synthesized as an intermediate of the glycolytic pathway in tissues and
is an important regulator of oxygen binding to hemoglobin. It binds hemoglobin in a
pocket formed by the two beta chains (see diagram). This pocket contains positively
charged amino acids that bind to the negatively-charged phosphate atoms in 2, 3
DPG, resulting in ionic bonds connecting each of the beta hemoglobin subunits to 2,
3-DPG. 2, 3-DPG only binds to deoxyhemoglobin; it does not bind to oxyhemoglobin.
Fetal hemoglobin (HbF) is synthesized during fetal development. It consists of the
usual two alpha chains and two gamma chains in place of beta chains. Within a few
weeks of conception, the fetal liver begins to form fetal hemoglobin. About 6O% of
the hemoglobin circulating during the last few months in utero and the first few
weeks of postnatal life in an infant is HbF. The affinity of HbF (Choice D) to oxygen is
much higher than that of HbA because HbF does not bind effectively to 2, 3-DPG.
This altered affinity is due to the presence of serine instead of charged histidine
residues at the normal 2, 3-DPG binding site. Fetal hemoglobin needs to bind more

224

USMLE WORLD STEP 1

BIOCHEMISTRY

avidly than adult hemoglobin because the fetal hemoglobin must obtain oxygen from
the mothers adult hemoglobin in the placenta.
(Choice A) HbAlc is formed by non-enzymatic glycosylation (attachment of glucose)
of HbA. In nondiabetic individuals, approximately4-6% of hemoglobin is glycosylated
while in patients with uncontrolled diabetes, the percentage of glycosylated
hemoglobin is much higher. HbAlc is used clinically to assess overall blood sugar
control for the three months preceding the test. Glycosylated hemoglobin does not
alter the binding of hemoglobin to 23- DPG.
(Choice B) HbS is the predominant form of hemoglobin in sickle cell disease. A
molecule of HbS contains two normal alpha chains and two mutated beta globin
chains characterized by replacement of glutamate at position six by valine.
(Choice C) HbC results from a mutation in the beta globin chain causing glutamic
acid to be replaced by lysine. These patients have mild, chronic hemolytic anemia.
(Choice E) A defect in the synthesis of alpha chains results in alpha thalassemia.
Normal individuals have four copies of the alpha gene, so several levels of alphaglobin defects can occur.
Educational Objective:
Normally, 2, 3-DPG forms ionic bonds with the two beta subunits of HbA in the
tissues after hemoglobin has been deoxygenated. Fetal hemoglobin binds oxygen
with a higher affinity due to its inability to interact with 2, 3- DPG. Ultimately, the
fetal hemoglobin must be able to extract 02 from maternal hemoglobin in the
placenta.

225

USMLE WORLD STEP 1

BIOCHEMISTRY

Q NO 26: A group of investigators discovers a potassium channel abnormality found


in pancreatic beta cells. This abnormality causes hypoglycemia because it makes the
potassium channels highly sensitive to their normal, activity-modulating, regulatory
substance. Which of the following regulatory substances is most likely implicated in
this case?
A. Glucose
B. Fructose-6-phosphate
C. Lactate
D. Pyruvate
E. Citrate
F. Malate
G. Fumarate
H. ATP

226

USMLE WORLD STEP 1

BIOCHEMISTRY

Explanation:
The regulation of insulin secretion from pancreatic beta cell is a complex process.
Glucose is the most important stimulator of insulin release. Glucose enters beta cells
by facilitated diffusion using glucose transporter 2 (GLUT 2) Glucose after entering
beta cells undergoes sequential oxidative metabolism through glycolysis, followed by
the citric acid cycle which generates ATP. A high ATP to ADP ratio within beta cells
causes the closure of potassium (KATP) channels. This closure occurs when ATP
binds to the regulatory subunit of the KATP channel. KATP channels are responsible
for the outward movement of potassium from the beta cells. When these channels
close, beta cells depolarize which results in the opening of voltage-dependent
calcium channels. High intracellular calcium then leads to insulin release. Defects of
the KATP channel gene can result in type two diabetes. The defect causes excess
function of this RATP channel; the channels do not close beta cells fail to be
depolarized and there is an inadequate insulin response to glucose. Sulfonylureas
are antidiabetic medications that are taken orallythey work by directly binding to
the regulatory subunits of KATP channels causing them to close.
(Choices B, C and D) Fructose 6-phosphate, pyruvate and lactate are intermediate
products of the glycolytic pathway. Lactate is produced by anaerobic glycolysis. Beta
cells have a rich oxygen supply and most of the glycolysis that occurs in beta cells is
aerobic. Thus a very small amount of lactate is produced. Pyruvate is converted into
acetyl co-enzyme A, which enters the mitochondria for metabolism via the citric acid
cycle.
(Choices E, F and G) Citrate malate, and fumarate are intermediates in the citric acid
cycle. During this process energy is stored as NADH and FADH2. ATP is then
produced when NADH and FADH2 enter oxidative phosphorylation. The products and
intermediates of glycolysis and the citric acid cycle listed in these choices do not
have any direct effect on KATP channels.
Educational Objective:
ATP is the regulatory substance that stimulates KATP channel closure in insulinproducing pancreatic beta cells.

227

USMLE WORLD STEP 1

BIOCHEMISTRY

Q NO 27: Skin fibroblasts incubated with radioactive amino acids synthesize


polypeptide chains that assemble to form a triple helix. Which of the
following amino acids is most avidly consumed by the fibroblasts?
Lysine
B. Proline
C. Alanine
D. Glycine
E. Leucine
F. Cysteine

A.

Explanation:
Collagen is the most abundant protein in the human body. The collagen molecule
consists of three polypeptide alpha chains held together by hydrogen bonds to form
a rope-like triple helix. Collagen is synthesized in fibroblasts, osteoblasts, and
chondroblasts. The collagen molecule has a triple helical conformation because each
of the alpha chains has a simple, repetitive amino acid sequence in which glycine
(Gly) appears at every third amino acid position. The amino acid sequence of
collagen is, therefore, designated (-Gly-X-Y-) 333. To fold into a triple helix, every
third amino acid in each alpha chain must be glycine. Glycine is the smallest amino
acid and it fits into the restricted space that exists when three alpha chains come
together to form a triple helix.
Many of the amino acids designated X and Y are proline residues. Proline is essential
for the formation of the alpha helix because its ring structure introduces a bend into
the polypeptide chains. Other X and Y amino acids form clusters of hydrophobic and
charged regions on the surface of the molecule that enables cross linking to other
collagen molecules. Lysine is commonly used in collagen for this purpose.
Educational Objective:
Glycine is the most abundant amino acid in the collagen molecule. It occurs in AT
LEAST every third amino acid position. The amino acid formula of collagen is (-Gly-XY-) 333.

228

USMLE WORLD STEP 1

BIOCHEMISTRY

Q NO 28: A newborn experiences lethargy, vomiting, and hypotonia during the first
few days of life. Laboratory examination reveals a metabolic acidosis with a large
anion gap, ketosis, and hypoglycemia. The concentration of propionic acid is
markedly increased in the plasma and urine. Metabolism of which of the following
amino acids contributes to this patients condition?
A. Phenylalanine
B. Valine
C. Asparagine
D. Histidine
E. Proline
F. Lysine

Explanation:
Catabolism of isoleucine, valine, threonine, methionine, cholesterol, and odd-chain
fatty acids leads to the formation of propionic acid, which is then converted to
methylmalonic acid by biotin-dependent carboxylation. Isomerization of
methylmalonyl CoA forms succinyl CoA, which then enters the TCA cycle. A
congenital deficiency of propionyl CoA carboxylase, the enzyme responsible for the
conversion of propionyl CoA to methylmalonyl CoA, leads to the development of
propionic acidemia, as propionyl CoA accumulates. Propionic acidemia is clinically
characterized by poor feeding vomiting, hypotonia, lethargy, dehydration, and an

229

USMLE WORLD STEP 1

BIOCHEMISTRY

anion gap acidosis. Propionic acid is the intermediate in the catabolism of branched
chain amino acids, such as valine, and is not produced during the catabolism of the
other amino acids listed.
(Choice A) Phenylalanine is converted to tyrosine by the enzyme phenylalanine
hydroxylase, which is defective in phenylketonuria (PRU).
(Choice C) Asparagine is a nonessential amino acid that is catabolized initially to
aspartate by the enzyme asparaginase. In rapidly dividing leukemic cells, the
synthesis of asparagine is impaired, so these cells survive by collecting asparagine
from circulating plasma. L-asparaginase works as an antineoplastic agent by
lowering circulating asparagine levels.
(Choice D) Histidine is an essential amino acid in children. Histidine is deaminated to
urocanic acid, which is then converted to N-forminino-glutamate (FIGIu). The
forminino group of FIGIu is donated to tetrahydrofolate to form glutamate and
forminino-tetrahydrofolate. Oxidative decarboxylation of histidine forms histamine,
which is released by mast cells in Type I Hypersensitivity reactions.
(Choice E) Proline is a nonessential amino acid that is oxidized to glutamate, which
in turn is transaminated to alpha ketoglutarate.
(Choice F) Lysine is an essential amino acid that is strictly ketogenic. The
metabolism of lysine is unique compared with the other amino acids, as it is not
transaminated as an initial step.
Educational Objective:
Propionyl CoA is derived from amino acids (Val, lie Met, and Thr), odd-numbered
fatty acids, and cholesterol side chains. Congenital deficiency of propionyl CoA
carboxylase, the enzyme responsible for the conversion of propionyl
CoA to methylmalonyl CoA, leads to the development of propionic acidemia.

230

USMLE WORLD STEP 1

BIOCHEMISTRY

Q NO 29: An agent applied to human cells is believed to activate G-protein


dependent phospholipase C. Which of the following intracellular substances
is most likely to increase immediately after exposure to this agent.
cAMP
B. Ca2+
C. Cl
D. NO
E. mRNA
F. cGMP

A.

Explanation:
A variety of hormones are known to exert their intracellular effects via the 1P3
second messenger system. A few of these hormones and receptors include alpha-i
adrenergic receptors oxytocin, the vascular receptor for vasopressin, angiotensin II,
growth hormone releasing hormone, thyroid releasing hormone and gonadotropin
releasing hormone. The IP3 second messenger system begins with the binding of
hormone to its cell surface receptor. This binding causes the exchange of GDP for
GTP on the alpha subunit of the G-protein associated with this receptor. The
activated alpha subunit undergoes a conformational change and exposes a
phospholipase C activating catalytic site. Phospholipase C is then activated.
Phospholipase C degrades phospholipids to diacylglycerol and inositol triphosphate
(1P3). Diacylglycerol is able to directly stimulate protein kinase C, but the major
activator of protein kinase C in this pathway is the increased intracellular calcium
that occurs when IP3 causes release of intracellular calcium stores from the

231

USMLE WORLD STEP 1

BIOCHEMISTRY

endoplasmic reticulum into the cytosol. Protein kinase C is the major affector
molecule in this pathway in that is directly modulated the activity of the enzymes
regulated by the hormone.
(Choices A and F) Intracellular cAMP and cGMP concentrations occur during
activation of the G-protein! adenylate cyclase or G-protein / guanylate cyclase
second messenger systems or in states where specific phosphodiesterases are
inhibited as can be seen upon exposure to phosphodiesterase inhibitors such as the
xanthine class of drugs and in the action of sildenafil, which selectively inhibits cGMP
phosphodiesterase.
(Choice C) The intracellular chloride (Cr) concentration increases in states of
neuronal inhibition by hyperpolarization after inhibitory neurotransmitters such as
GABA or glycine act on the neuron and increase Cl- conductance.
(Choice D) Intracellular nitric oxide (NO, endothelium-derived relaxing factor) is
increased in states where vasodilatation is desired by the body and also in the
production of a penile erection. NO is synthesized from arginine and oxygen by the
enzyme nitric oxide synthase.
(Choice E) The intracellular concentration of mRNA increases during high cellular
states of protein synthesis such as during division.
Educational Objective:
The IP3 second messenger system begins with hormone binding and G-protein
activation leading to activation of phospholipase C. Phospholipase C forms
diacylglycerol and IP3 from phospholipids, and IP3 causes an increase in intracellular
calcium, which then activates protein kinase C.

232

USMLE WORLD STEP 1

BIOCHEMISTRY

Q NO 30: It has been determined in healthy volunteers that the chloride content of
erythrocytes is much lower in arterial blood than in venous blood. The action of
which at the following is mostly responsible for the observed difference?
A. Spectrin
B. Na/K ATPase
C. Carbonic anhydrase
D. 2, 3-biphosphoglycerate mutase
E. Glucoso-6-phosphate dehydrogenase

Explanation:
Hemoglobin is only found within red blood cells (RBCs), and is responsible not only
for oxygen delivery to the tissues, but also for carrying carbon dioxide from the
tissues to the lungs. Hemoglobin carries carbon dioxide as carbamate, which is
formed as follows:
CO2 + Hb-NH3 = 2H+ + Hb-NH-CO2
Hemoglobin is responsible for carrying l5% of carbon dioxide as carbamate and the
remainder of carbon dioxide is carried as bicarbonate ion within the red blood cells.
Carbon dioxide produced by tissue respiration enters red blood cells and is hydrated
by carbonic anhydrase to form carbonic acid. Carbonic acid then undergoes
spontaneous conversion to HC03 ad Ht Many of the bicarbonate ions diffuse out of
the RBC into the plasma. To maintain the electrical neutrality chloride ions diffuse
into the REC to take their place. This process is called chloride shift and it is the
principal cause of high REC chloride content in venous blood.
(Choice A) Spectrin is a structural component of the membrane of RBC5 that gives
the cells flexibility. Abnormalities of spectrin result in abnormal red blood cell shape
and susceptibility to hemolysis.
(Choice B) Na+/K+-ATPase is an important ion pump responsible for the
maintenance of ionic concentration gradients across the plasma membrane. This
energy-requiring pump is electrogenic in nature because it extrudes three sodium

233

USMLE WORLD STEP 1

BIOCHEMISTRY

ions for every two potassium ions that enter the cell, which creates negative
intracellular potential. This pump is not responsible for movement of chloride ions.
(Choice D) 2, 3-DPG is present in small quantities in most cells; however, in
erythrocytes the concentration of 2, 3- DPG is high. In the RBCs, 2-3-DPG combines
with hemoglobin and decreases its affinity for oxygen, which facilitates oxygen
delivery to the tissue. 2, 3-DPG itself is not responsible for the chloride shift.
(Choice E) Clucose-6-phosphate dehydrogenase is the first enzyme in the pentose
phosphate pathway. Patients with glucose-6-phosphate dehydrogenase deficiency
present with episodic hemolysis induced by oxidant stressors.
Educational Objective:
Carbonic anhydrase activity within erythrocytes forms bicarbonate from CO2 and
water. Many of the bicarbonate ions diffuse out of the RBC into the plasma. To
maintain the electrical neutrality chloride ions diffuse into the RBC to take their
place. This process is called chloride shift and it is the principal cause of high RBC
chloride content in venous blood.

234

USMLE WORLD STEP 1

BIOCHEMISTRY

Q NO 31: DNA exonucleases hydrolytically remove one nucleotide at a time from


the end of a DNA chain. Which of the following enzymes has 5 to 3 exonuclease
activity?
A. Helicase
B. Primase
C. Gyrase
D. DNA polymerase III
E. DNA polymerase I
F. Ligase

Explanation:
DNA polymerases are the primary enzymes responsible for DNA replication. In E.
coil, there are three major DNA polymerases: DNA polymerase I, II, and III. DNA
replication requires a high degree of fidelity in order to preserve the genetic code in
daughter cells and prevent potentially lethal mutations. This high fidelity replication
is accomplished by the 3 to 5 proofreading exonuclease activity of DNA
polymerase. (Only DNA polymerase I has 5 to 3 exonuclease activity). This 5 to 3
exonuclease activity of DNA polymerase I functions to remove the RNA primer (3hydroxyl group) which is used by DNA polymerase III for the initiation of DNA
replication. The 5to 3 exonuclease activity of DNA polymerase I also performs
exonuclease excision and repair of damage to parent DNA.
(Choice A) Before the process of DNA replication begins, the parent strand of DNA
unwinds and dissociates secondary to the action of the enzyme helicase.
(Choice C) The enzymes topoisomerase I and II release tension caused during DNA
strand unwinding by relieving both negative and positive supercoils in eukaryotic
cells. The enzyme topoisomerase II, also known as DNA gyrase in prokaryotic cells,
has a slightly different function than the eukaryotic topoisomerase II and does not
have 5 to 3 exonuclease activity.
(Choice B) Once a strand is unwound, DNA polymerases begin to synthesize
complementary strands in the 5 to 3 direction, but they require a free 3-hydroxyl
group primer (RNA primer). This free 3-hydrorl group is placed on the strand to be
duplicated by an RNA polymerase called primase.
(Choice F)The Okazaki fragments of the lagging strand are bound together by the
enzyme ligase.
Educational Objective:

235

USMLE WORLD STEP 1

BIOCHEMISTRY

DNA polymerase I has 5 to 3 exonuclease activity in addition to its 5 to 3


polymerase and 3 to 5 exonuclease activities. This 5 to 3 exonuclease activity is
used to remove the RNA primer (which initiates DNA polymerization) and to remove
damaged DNA.

236

USMLE WORLD STEP 1

BIOCHEMISTRY

Q NO 32: A 32-year-old male suffers from a chronic cough and weight loss. M.
tuberculosis bacteria isolated from his sputum are effectively inhibited by lowdose streptomycin grown on artificial media. Which of the following steps in
bacterial protein synthesis is most likely inhibited by this antibiotic?
mRNA synthesis
B. Aminoacyl-tRNA binding
C. Initiation
D. Peptide bond formation
E. Translocation

A.

Explanation:
Protein synthesis involves several steps, beginning with mRNA formation from a DNA
template and ending with the formation of proteins in the cytoplasm. Translation is
the process by which the information carried by mRNA is converted into proteins on
ribosomes, which also requires tRNA and multiple other factors for the initiation and
elongation of the polypeptide chain. The initiation of translation in prokaryotes
requires a specific N-formulated methionine-tRNA that is used to incorporate the
initial methionine residue into all proteins. In prokaryotic mRNA, the initiation codon
AUGI also the codon for methionine, is located six to ten bases downstream from a
genetic sequence known as the Shine-Dalgarno element, which recognizes
complementary sequences in the 16S rRNA and the 30S small ribosomal subunit.
The Shine-Dalgarno element is unique to prokaryotes, as eukaryotes do not possess
the sequence.
Remember that ribosomal subunits consist of both rRNA and ribosomal proteins.
Ribosomes are made up of two subunits in both prokaryotes (30S and 50S) and

237

USMLE WORLD STEP 1

BIOCHEMISTRY

eukaryotes (60S and 40S), differing primarily in structure. Binding of the 30S
ribosomal subunit to mRNA and an N-formylated methionine-tRNA forms the
prokaryotic initiation complex, facilitated by the initiation factors IF1, IF2, and IF3.
The energy for this reaction is obtained through the hydrolysis of GTPI which also
brings the 50S ribosomal subunit to the initiation complex, resulting in formation of
the 70S ribosomal unit.
Thus, the combination of the 50S and the 30S ribosomal subunits forms the 703
complete ribosomal complex. Streptomycin binds the 30S ribosomal subunit and
distorts its structure, impairing the initiation of protein synthesis and inhibiting
formation of the initiation complex. Since the ribosomal structures in prokaryotes
and eukaryotes differ, this difference can be exploited in the production of
antibiotics, which selectively inhibit protein synthesis in prokaryotes, while not
affecting mammalian cells.
(Choice D) Ribosomes have three sites for attachment of tRNA molecules: the A, P,
and E sites. During translation, the A site binds an incoming aminoacyl-tRNA, which
is then transferred to the P site after incorporation into the peptide chain. The amino
acid is subsequently cleaved from the tRNA molecule, and the empty tRNA is then
shifted to the E site. Peptidyltransferase is a component of the 50S ribosomal
subunit necessary for the transfer of amino acids from the A to the P site and for the
formation of peptide bonds. Chloramphenicol is an antibiotic that inhibits this
peptidyltransferase enzyme.
(Choice E) During elongation, ribosomes move in the 5 to 3 direction on the mRNA
molecule in a process known as translocation. The antibiotics clindamycin and
erythromycin both inhibits this translocation step.
(Choice B) Tetracycline antibiotics interfere with the binding of aminoacyl-tRNA to
the A site.
Educational Objective:
Streptomycin inhibits the initiation of protein synthesis by binding to and distorting
the structure of the prokaryotic 3OS ribosomal subunit.

238

USMLE WORLD STEP 1

BIOCHEMISTRY

Q NO 33: An infant born to 22-year-old female experiences lethargy vomiting and


jaundice soon afterbirth. The infant is placed on a galactose-free formula and
shows gradual improvement. Which of the following steps in galactose metabolism
is most likely impaired in this patient?

A.
B.
C.
D.
E.

A
B
C
D
E

Explanation:
The patient described in the vignette has characteristic features of galactosemia.
Classic galactosemia is caused by impaired galactose-i-phosphate metabolism.
Lactose degradation by the intestinal disaccharidase lactase leads to the formation of
galactose and glucose; this enzyme is defective in lactose deficiency. Galactose is
then phosphorylated to galactose-i-phosphate by the enzyme galactokinase.
Galactose-1-phosphate is then converted to glucose-i-phosphate by epimerization.
This reaction requires the transfer of uridine diphosphate (UDP) from UDPglucose
catalyzed by galactose-1-phosphate uridyl transferase (SALT) generating UDP
galactose and glucose-1-phosphate. UDP-galactose is then epimerized to UDPglucose by UDP-galactose-4 epimerase. In breast tissue a -1, 4 glycosidic linkages

239

USMLE WORLD STEP 1

BIOCHEMISTRY

between glucose and galactose is formed resulting in galactosyl -1, 4 glucose


(lactose or milk sugar).
Galactosemia can result from defects in any of the three enzymes involved in
galactose metabolism: however. The most common form of galactosemia, classic
galactosemia, occurs from deficiency of galactose-1-phosphate uridyl transferase.
The clinical features of this illness include vomiting lethargy and failure to thrive
soon after breast feeding is begun. Galactosemia can result in impaired liver
function, hyperchloremic metabolic acidosis, and aminoaciduria. Dietary restriction of
lactose results in improvement in symptoms.
Excess galactose in patients with galactosemia is converted to galactitol by galactose
reductase or to galactonic acid by galactose oxidase. While galactonic acid can be
metabolized by the HMP shunt galactitol accumulates within the cells. Untreated
galactosemia typically culminates in irreversible eye and liver damage. Galactokinase
deficiency typically causes less severe manifestations with cataract being the most
common manifestation.
Educational Objective:
Classic galactosemia results from deficiency of galactose-1-phosphate uridyl
transferase; this defect is the most common cause of galactosemia. The clinical
features of this illness include vomiting lethargy and failure to thrive soon after
breast feeding is begun.

240

USMLE WORLD STEP 1

BIOCHEMISTRY

Q NO 34: An infant born to a 26-year-old Caucasian female demonstrates poor


feeding and developmental delay. Laboratory evaluation reveals megaloblastic
anemia and high plasma homocysteine levels. Based on laboratory findings, itis
suggested that the patient is unable to form methylcobalamin. Synthesis of
which of the following substances is most likely impaired in this patient?
Cystathionine
B. 5-adenosylmethionine
C. Methionine
D. Cysteine
E. Methylmalonyl CoA

A.

Explanation:
The metabolism of methionine first involves conversion of methionine to S-adenosylmethionine (SAM) by the enzyme S-adenosyl-methionine synthetase. SAM has an
activated methyl group that can be transferred to a variety of acceptor molecules
forming methylated products. S-adenosyl-methionine is converted to S-adenosylhomocysteine after transfer of its activated methyl group, and S-adenosylhomocysteine is then catabolized to form adenosine and homocysteine. Conversion
of homocysteine to cystathionine requires the enzyme cystathionine synthetase,
Vitamin B6 as a cofactor, and the amino acid serine. A deficiency of or defect in
cystathionine synthetase is the most common cause of homocystinuria.
Cystathionine is then converted to cysteine in a reaction catalyzed by cystathionase,
also requiring Vitamin B6 as a cofactor.
Alternatively homocysteine can be converted back to methionine by combining with
N-5-methyl-tetrahydrofolate in a reaction requiring vitamin B12 (cobalamin). In this

241

USMLE WORLD STEP 1

BIOCHEMISTRY

reaction, a methyl group is transferred from a methylated tetrahydrofolate to


homocystine, forming methionine and tetrahydrofolate. Because vitamin B12 is
required for the conversion of N-5-methyl-tetrahydrofolate (N5met-THE)to
tetrahydrofolate (THE), vitamin B12 deficiency results in accumulation of N5met-THF
and a decrease in THE levels, in a process known as the folate trap. Patients with
cobalamin deficiency classically develop megaloblastic anemia and homocystinemia
as methionine re-synthesis is impaired (Choice C).
(Choices A and B) The formation of SAM and cystathionine do not require cobalamin
and therefore synthesis of these molecules are not impaired in B12 deficiency.
Vitamin B5, however, is required by cystathionine synthase to form cystathionine.
(Choice D) Cysteine synthesis is impaired in patients with classic homocystinuria,
but not in patients with cobalamin deficiency.
(Choice E) The conversion of methylmalonyl COA to succinyl COA is catalyzed by the
enzyme methylmalonyl COA mutase, which requires vitamin B12 as a coenzyme.
Thus, levels of methylmalonyl COA will increase in Vitamin B12 deficiency.
Educational Objective:
Cobalamin (Vitamin B12 deficiency results in homocystinemia due to impaired
methionine synthesis.
Homocystinuria occurs in cobalamin (Vitamin B12 deficiency because homocysteine
methyltransferase, the enzyme that converts homocysteine and
methyltetrahydrofolate to methionine and tetrahydrofolate, requires B12 as a
cofactor.

242

USMLE WORLD STEP 1

BIOCHEMISTRY

Q NO 35: The rate of glycogen degradation in skeletal muscles increases several


hundred-fold after the onset of contraction compared to the resting state. This
finding reflects enzyme activation by which of the following substances?
A. ADP
B. cAMP
C. Ca2+
D. Lactate
F. Glucose-6-phosphate

Explanation:
Glycogen in skeletal muscle is used exclusively within the muscle for energy during
exercise. Skeletal muscle glycogen stores do not contribute to blood glucose levels
during fasting. Glycogen is broken down by the enzyme glycogen phosphorylase.
Regulation of glycogen synthesis and breakdown is a complex process and depends
on activation and deactivation of glycogen phosphorylase and glycogen synthase,
which is regulated by a number of glycolysis intermediates, calcium, cyclic AMP, and
noncyclic AMP.
During muscle contraction, calcium is released into the cytosol from the sarcoplasmic
reticulum. In addition to binding troponin C and activating muscle contraction, this
calcium activates myophosphorylase (muscle glycogen phosphorylase) to cause
breakdown of glycogen and release of glucose 1-phosphate (choice C), which is then
converted to glucose 6-phosphate by an enzyme called phosphoglucomutase.
Muscles cannot release glucose into the circulation because they lack the enzyme
glucose 6-phosphatase required to convert glucose-6-phosphate to free glucose.
Glucose 6-phosphate can then enter the glycolytic pathway to provide energy for
contracting muscles.

243

USMLE WORLD STEP 1

BIOCHEMISTRY

(Choices A and B) Glucagon and epinephrine by acting on G-protein-coupled


receptors on the hepatic cell membrane lead to cleavage of AlP forming cyclic AMP,
which in turn stimulates protein kinase A. Phosphorylation of glycogen
phosphorylase by protein kinase A leads to activation, whereas, phosphorylation of
glycogen synthase by this enzyme leads to its inactivation. There is a fine balance
between phosphorylation by protein kinase A and dephosphorylation by
phosphatase, and this maintains the activity of the phosphorylase and synthase
enzymes at proper levels for the needs of the tissue. Under extreme anaerobic
conditions, AMP stimulates the myophosphorylase enzyme without its
phosphorylation. In contracting muscles, calcium also activates myophosphorylase
without its activation by phosphorylation. Both epinephrine and glucagon increase
glycogen breakdown in the liver, but only epinephrine (not glucagon) has significant
effect on glycogen breakdown in muscles.
(Choice D) Lactate is produced in tissues during anaerobic glycolysis. The classic
example of this is the production of lactic acid by muscles during very strenuous
exercise leading to relatively hypoxic conditions in the muscles. The lactate produced
by the muscles can then be converted to glucose in the liver by the process of
gluconeogenesis. In contrast to the muscles, the liver can release glucose after
glycogen breakdown because the liver expresses glucose6-phosphatase.
(Choice E) Glucose 6-phospate activates the glycogen synthase enzyme and
inactivates glycogen phosphorylase while calcium directly stimulates glycogen
phosphorylase.
Educational Objective:
Glycogen degradation is coupled with skeletal muscle contraction due to calciummediated myophosphorylase activation. Muscle is stimulated to contract by the
neurotransmitter acetylcholine at the neuromuscular junction. This stimulation
causes depolarization of the muscle cell and release of calcium from the
sarcoplasmic reticulum into the cytosol. Increased calcium in the cytosol binds
troponin C to cause muscle contraction, and it also activates muscle glycogen
phosphorylase to provide energy for the working muscle.

244

USMLE WORLD STEP 1

BIOCHEMISTRY

Q NO 36: A 6-year-old male with hemolytic anemia is found to have an abnormality


due to an inactive erythrocyte enzyme. The defective enzyme contains 156 amino
acid residues instead of the normal 190 residues. A point mutation in exon 2 of the
enzyme gene is identified as the cause for this patients disease. Which of the
following mRNA code changes is most likely in this case?
A. UAAUAG
B. UUUUUA
C. CUUAUU
D. UCAUGA
E. UAC--CAC
Explanation:
Messenger RNA (mRNA) is produced from DNA by RNA polymerase II. After
processing, which includes removal of noncoding regions of mRNA (introns) the
mRNA molecule is transported to the cytoplasm for translation. Remember that the
mRNA molecule is composed of groups of three sequential bases known as codons.
Since there are 64 possible combinations of the four bases found in DNA, there are
64 possible codons. Because there are only twenty amino acids, most amino acids
have more than one potential codon. For instance, UUU and UUA both code for the
amino acid phenylalanine. Some codons call for the termination of synthesis of the
polypeptide chain and are referred to as stop codons, including UAA, UAG, and UGA.
Changes in the DNA sequence (mutations) can result in alterations of protein
structure, affecting enzyme function. In the case described in the question stem the
structure of the protein is significantly shortened secondary to an alteration of the
genetic code through a point mutation, which most likely resulted in the introduction
of a premature stop codon. Of the choices listed the single base change of UCA
(serine) to USA results in premature termination of protein synthesis and the
formation of a truncated protein molecule. The introduction of a stop codon in the
middle of a protein sequence is called a nonsense mutation (Choice D).
(Choices A and B) Changing UAA to UAG would not modify the structure of the
protein because both of these are stop codons. This type of mutation is called a
silent mutation. Similarly UUU and UUA both code for phenylalanine and this
mutation will not alter the protein structure either, as itis also a silent mutation.
(Choices C and E) Changing CUU (leucine) to AUU (isoleucine) will result in an amino
acid change at one position. The function of this protein maybe altered depending on
a variety of factors, but the ultimate size of the protein will remain the same. This
type of mutation is called a missense mutation.
Educational Objective:
USIA, UAGI and UAA are stop codons, and mutations producing abnormally placed
stop codons are called nonsense mutations.

245

USMLE WORLD STEP 1

BIOCHEMISTRY

Q NO 37: Liver cells that demonstrate a high concentration of fructose 2, 6biphosphate have a low rate of which of the following conversions?
A. Glucose glycogen
B. Acet CoA fatty acids
C. Alanine glucose
D. Pyruvate oxaloacetate
E. a-Ketoglutarate glutamate

Explanation:
Gluconeogenesis is not simply the reverse of glycolysis; three of the ten enzymes in
glycolysis are unidirectional. These enzymes are hexokinase (glucokinase),
fructokinase, and pyruvate kinase. To reverse glycolysis, that is, to form glucose
from pyruvate, four different enzymes are required to overcome these roadblocks
created by the three unidirectional enzymes of glycolysis. These four enzymes in
gluconeogenesis are pyruvate carboxylase, phosphoenolpyruvate carboxykinase,
fructose, 6-bisphosphatase, and glucose-6-phosphatase. The first two enzymes
convert pyruvate to phosphoenolpyruvate via oxaloacetate. The conversion of
fructose 1, 6- bisphosphate to fructose-6-phosphate is achieved by the enzyme
fructose-1, 6-bisphosphatase. This enzyme is allosterically inhibited by fructose 2, 6bisphosphate; therefore, high concentrations of fructose 2, 6-bisphosphate inhibit
gluconeogenesis and promote glycolysis.
By inhibiting gluconeogenesis, high concentrations of fructose 2, 6-bisphosphate will
also decrease the conversion of alanine to glucose. The inter conversion of fructose6-phosphate and fructose 2, 6-bisphosphate is achieved by a bifunctional enzyme
composed of phosphofructokinase-2, which converts fructose-6-phosphate to
fructose 2,6- bisphosphate, and fructose 2,6-bisphosphosphatase, which converts

246

USMLE WORLD STEP 1

BIOCHEMISTRY

fructose 26-bisphosphate to fructose-6- phosphate. Glucagon causes


phosphorylation of this enzyme leading to inactivation of the kinase and activation of
the phosphatase. Insulin causes dephosphorylation leading to activation of the
kinase part of this bifunctional enzyme. Thus glucagon decreases the level of
fructose 26-bisphosphate causing release of allosteric inhibition on the enzyme
fructose, 6-bisphosphatase and augmentation of gluconeogenesis.
(Choice A) Glycogen is the main storage form of glucose in animals. Glycogen is
primarily stored in the liver and muscles. During the fed state, glucose residues are
added to preexisting glycogen chains. Elevation of both insulin levels as well as
glucose concentrations is required to stimulate the formation of glycogen.
(Choice D) Conversion of pyruvate to oxaloacetate is achieved by the enzyme
pyruvate carboxylase. This enzyme is allosterically stimulated by acetyl CoA. When
high levels of acetyl CoA are available. Pyruvate is preferentially converted to
oxaloacetate. Fructose 26-bisphosphate does not have effect on conversion of
pyruvate to oxaloacetate.
(Choice E) Transamination is not altered by the level of fructose 2,6 bisphosphate.
Educational Objective:
Fructose 2, 6-biphosphate activates glycolysis by inducing PEK-1 and inhibits
gluconeogenesis by inhibiting fructose 1,6-biphophatase.

247

USMLE WORLD STEP 1

BIOCHEMISTRY

Q NO 38: If energy-dependent organic anion transport across the hepatocellular


membrane is selectively inhibited which of the following would most likely result?
A. Decreased haptoglobin concentration in the serum
B. Increased urobilinogen absorption in the gut
C. Increased bilirubin excretion in the urine
D. Decreased indirect bilirubin uptake from the blood
E. Increased stercobilinogen excretion in the feces

Explanation:
The liver takes up indirect (unconjugated) bilirubin through a passive process and
secretes direct (conjugated) bilirubin through an active process. Inhibition of active
secretion of conjugated bilirubin into the bile canaliculi results in an isolated
conjugated hyperbilirubinemia. Both unconjugated bilirubin and bilirubin
glucuronides may accumulate systemically and deposit in tissues, giving rise to the
yellow discoloration of jaundice. This yellow discoloration is particularly evident in
the sclerae (icterus). There are two important pathophysiologic differences between
the two forms of bilirubin. Unconjugated bilirubin is virtually insoluble in water at
physiologic pH and is tightly complexed to serum albumin while in the circulation S
This form cannot be excreted in urine even when blood levels are high S In contrast,
conjugated bilirubin is water soluble nontoxic, and only loosely bound to albumin.

248

USMLE WORLD STEP 1

BIOCHEMISTRY

Because of its solubility and weak association with albumin, excess conjugated
bilirubin in plasma can be excreted in urine.
Jaundice occurs when the equilibrium between bilirubin production and clearance is
disturbed by one or more of the following mechanisms: (1) excessive production of
bilirubin, (2) reduced hepatocyte uptake, (3) impaired conjugation, (4) decreased
hepatocellular excretion, and (5) impaired bile flow (both intrahepatic and
extrahepatic). The first three mechanisms produce unconjugated hyperbilirubinemia,
and the latter two produce predominantly conjugated hyperbilirubinemia.
(Choice A) A decreased haptoglobin concentration in the serum would be expected in
cases of hemolytic anemia such as intravascular hemolysis (ABC incompatible
transfusion reactions). During hemolysis, a large amount of free iron is released into
the blood and is bound by haptoglobin molecules, which are then removed from
circulation by the reticuloendothelial system (spleen and liver). Thus, it would
produce more of the unconjugated bilirubin.
(Choice B) Increased urobilinogen absorption in the gut is incorrect because as the
biliary secretion of conjugated bilirubin is inhibited, as in this case, less bilirubin is
excreted into the GI tract, and therefore less bilirubin is available to be converted to
urobilinogen and reabsorbed.
(Choice 0) Decreased indirect bilirubin uptake from the blood is incorrect because
the liver takes up indirect bilirubin through a passive process, not an active process
as mentioned in the question stem.
(Choice E) Increased stercobilinogen excretion in the feces would not occur in states
where the biliary excretion of conjugated bilirubin is inhibited, as in this case,
because in these states less bilirubin is excreted into the GI tract.
Educational Objective:
The liver takes up indirect (unconjugated) bilirubin through a passive process and
secretes direct (conjugated) bilirubin through an active process. Unconjugated
bilirubin is virtually insoluble in water at physiologic pH and is tightly complexed to
serum albumin while in the circulation. This form cannot be excreted in the urine
even when blood levels are high.

249

USMLE WORLD STEP 1

BIOCHEMISTRY

Q NO 39: The oxygen dissociation curve for hemoglobin is presented on the slide
below. During the transition from point 1 to point 2 on this curve, the
hemoglobin molecule is most likely to release which of the following?
A. Heme
B. Protons
C. Chloride
D. Oxygen
E. Phosphate

Explanation:
At a very low p02, the
hemoglobin molecule is
fully deoxygenated.
0xygen-binding affinity increases as the p02 increases primarily because there is an
abrupt increase in oxygen binding affinity after some oxygen has been bound to the
hemoglobin molecule. More O2 molecules bind as the oxygen partial pressure
increases; then, as hemoglobin becomes saturated with oxygen, very little additional
binding occurs, and the curve levels out. The transition from point to point 2 on the
above graph illustrates complete oxygen saturation of hemoglobin at a high p02,
which is what happens to hemoglobin in the lungs.

The release of oxygen from hemoglobin is enhanced at the tissue level by the
increased partial pressure of carbon dioxide and the resultant decrease in pH. This is

250

USMLE WORLD STEP 1

BIOCHEMISTRY

known as the Bohr Effect. This effect is caused by ionizable histidine side chains at
the N-terminal alpha amino group of the alpha and beta hemoglobin subunits that
bind protons with high affinity when hemoglobin is deoxygenated. The increased
concentration of protons at the tissue level causes the amino groups to be
protonated (charged) and able to form ionic bridges (see diagram). The formation of
these salt bridges stabilizes the deoxy form of hemoglobin and decreases
hemoglobins affinity for oxygen there by facilitating the release of oxygen at the
tissue level.
The opposite effect occurs at the level of the lung; the increase in p02 at the
alveolar level increases the binding of oxygen to hemoglobin and causes the release
of protons. The concentration of protons is greater at the tissue level because of
increased concentrations of CO2 in the tissues. CO2 released by the tissues is
converted by carbonic anhydrase to bicarbonate ions and hydrogen ions. Some
bicarbonate is carried by hemoglobin to the lungs while the remainder is transported
in its ionic form freely in the blood. In the lungs, carbonic anhydrase converts the
protons and bicarbonate ions back into CO2 and water.
(Choice A) Heme is not released from hemoglobin in the context of oxygen loading
or dissociation; it is however, released from hemoglobin during the normal
destruction of aged red blood cells by the spleen.
(Choices C and E) Neither chloride, nor phosphate is transported in any important
manner by hemoglobin. Chloride is shifted out of erythrocytes in venous blood by
bicarbonate, and it re-enters RBCs in the arterial blood.
(Choice D) Oxygen is released from hemoglobin in the tissues most commonly where
conditions favor proton binding and the release of in the tissues, the pH is lower, the
temperature is greater, and the concentration of 2, 3 DPG is greater than in the lung.
All of these conditions favor a decreased affinity of hemoglobin for O2.
Educational Objective:
In the lungs, hemoglobin binds oxygen and releases protons; while, in the tissues, it
releases O2, and acquires protons. Deoxyhemoglobin is stabilized by ionic bonding of
2, 3 DPG to the two beta subunits and by salt bridges between N-terminal histidine
residues in each globulin.

251

USMLE WORLD STEP 1

BIOCHEMISTRY

Q NO 40: Hepatocytes exposed to an external stimulus demonstrate a rapid


increase in intracellular glycogen stores and a decrease in glucose release
into the blood. Which of the following most likely promotes the effects
described above?
Protein phosphatase-1
B. Protein kinase A
C. Phospholipase C
D. Janus protein kinase (JAR)
E. Lipoxygenase

A.

Explanation:
The response of the hepatocytes to the stimulus described in the question stem is
characteristic of the response of these cells to insulin. Insulin is an anabolic hormone
that promotes the synthesis of glycogen, triacylglycerides, nucleic acids, and
proteins. Insulin inhibits glycogenolysis and gluconeogenesis. Insulin acts via a
tyrosine kinase mechanism. The insulin cell surface receptor is a transmembrane
protein that also has cytosolic tyrosine kinase activity. The tyrosine kinase causes
phosphorylation of a poorly characterized class of proteins known as insulin receptor
substrates leading to activation of protein phosphatase. Protein phosphatase
dephosphorylated glycogen synthase thereby activating that protein and promoting
glycogen synthesis. Protein phosphatase also dephosphorylated fructose 1 6bisphosphatase thereby inactivating that enzyme and inhibiting gluconeogenesis.
This is also a good example of how phosphorylation and dephosphorylation of
enzymes by second-messenger proteins can cause activation of some enzymes and
inactivation of others.
(Choice B) Protein kinase A is the primary intracellular affector enzyme in the Gprotein / adenylate cyclase second messenger system. Increased levels of cAMP
stimulate protein kinase A to activate the necessary enzymes to carry out the
intracellular actions of the hormone that bound the cell and activated adenylate
cyclase in the first place.
(Choice C) Phospholipase C is active in the G-protein / Inositol triphosphate (1P3) /
Calcium second messenger system. Hormone binds its receptor and activates a Gprotein that in turn activates phospholipase C to degrade phospholipids into inositol
triphosphate and diacylglycerol. Both diacylglycerol and the increased intracellular
calcium caused by IP3 will activate protein kinase C.
(Choice D) Janus protein kinase (JAR) is a part of the second messenger system for
peptide hormones such as some cytokines in a pathway referred to as JAR-STAT
(signal transducers and activators of transcription). JAR has tyrosine kinase activity.
(Choice E) Lipoxygenase is an enzyme involved in arachidonic acid metabolism and
is responsible for the arm of that pathway that synthesizes leukotrienes.
Educational Objective:
Insulin is an anabolic hormone that acts via a tyrosine kinase second messenger
system to stimulate the synthesis of glycogen, proteins, fatty acids and nucleic
acids. Tyrosine kinase leads to the activation of protein phosphatase within cells, and
protein phosphatase directly modulates the activity of enzymes in the metabolic
pathways regulated by insulin.

252

USMLE WORLD STEP 1

BIOCHEMISTRY

Q NO 41: A 14-year-old male is being evaluated for splenomegaly. Enzyme assays


performed on circulating blood cells demonstrate low pyruvate kinase activity.
Which of the following is the most likely cause of this patients splenomegaly?
A. Intracellular substance accumulation
B. Passive congestion
C. Inflammatory infiltration
D. Work hypertrophy
E. Neoplastic lesion
Explanation:
The spleen is an organ of the reticuloendothelial system that contains approximately
25% of bodys lymphoid tissue. One of the main functions of the spleen in adult
humans is maintenance of erythrocyte quality in the red pulp by removal of
senescent and defective red blood cells. The spleen accomplishes this function
through the unique organization of its parenchyma and vasculature. Antibody
production and B cell affinity maturation occur in the white pulp of the spleen, and
the spleen also serves to remove antibody-coated bacteria and other opsonized
material and cells from the circulation. An increase in any of these normal functions
may result in splenomegaly.
Pyruvate kinase is the enzyme in the glycolytic pathway that converts
phosphoenolpyruvate to pyruvate resulting in the generation of a molecule of ATP.
Pyruvate kinase is allosterically stimulated by fructose 1, 6-bisphosphate, which is
produced from fructose-6-phosphate by the enzyme phosphofructokinase. Allosteric
stimulation of pyruvate kinase by fructose 1, 6-bisphosphate results in stimulation of
glycolysis. Red blood cells do not contain mitochondria, so the main metabolite of
glycolysis is lactate. Any deficiency of glycolysis in red blood cells leads to hemolysis
because of insufficient production of ATP and defective maintenance of red blood cell
architecture. Excessive erythrocyte destruction by the spleen causes splenomegaly
due to work hypertrophy (choice D). Work hypertrophy results from hypertrophy of
the reticuloendothelial cells of the splenic parenchyma as these cells are involved in
the removal of damaged RBCs.
(Choices A, B, C and E) The other choices listed will also result in splenomegaly.
Accumulation of sphingomyelin and glucocerebrosides are responsible for
splenomegaly in Niemann-Pick disease and Gaucher disease, respectively (A).
Passive congestion of spleen occurs with portal hypertension, splenic vein
thrombosis and congestive heart failure (B). In these conditions, dilatation of the
splenic sinusoids leads to congestion of the spleen with blood resulting in
splenomegaly. The spleen is very important for fighting infectious pathogens in the
body: therefore, many acute and chronic infections lead to enlargement of the
spleen due to proliferation of lymphoid tissue (C). Disorders such as leukemia and
lymphoma also result in splenomegaly from neoplastic proliferation of lymphoid
tissue within the spleen (E).
Educational Objective:
Pyruvate kinase deficiency causes hemolytic anemia due to failure of glycolysis and
resultant failure to generate sufficient ATP to maintain erythrocyte structure. In this
case, splenic hypertrophy results from increased work of the splenic parenchyma,
which must remove these deformed erythrocytes from the circulation.

253

USMLE WORLD STEP 1

BIOCHEMISTRY

Q NO 42: An 18-year-old Caucasian male spends five hours in the Florida sun at a
local beach without applying any sunscreen to his skin. Which of the following is
most likely to happen with in the patients skin cells as a result of this exposure?
A. Double-strand DNA breaks undergo end-joining repair
B. Adjacent purine bases form a covalent bond
C. Specific glycosylase removes deaminated bases
D. Adenine residues of the undamaged strand become methylated
E. Specific endonuclease nicks the damaged DNA strand

Explanation:
DNA can be damaged by a number of agents including chemicals, ultraviolet
radiation, and ionizing radiation. By forming an abnormal covalent bond between
adjacent thymine residues, ultraviolet rays damage DNA through the formation of
thymine dimers. Secondary to this covalent bonding, transcription and replication
can not proceed normally. On the other hand thymine dimers can be removed by the
action of ultraviolet-specific endonucleases. Ultraviolet-specific endonucleases
specifically detect abnormalities in the DNA structure caused by the formation of
DNA photoproducts and then excise the defective region. This enzyme causes nicks
at damaged sites that are later excised by the 5 to 3 exonuclease activity of DNA
polymerase, which also synthesizes new DNA in the place of the damaged DNA. A
lack of the repair enzyme ultraviolet-specific endonuclease results in a lack of repair
of ultraviolet radiation induced thymine dimmers, a condition known as Xeroderma
Pigmentosum (XP).
(Choice A) Repair of double strand DNA fractures occurs following exposure to
ionizing radiation (X rays and gamma rays), which can be joined together by
nonhomologous or homologous re-ligation.

254

USMLE WORLD STEP 1

BIOCHEMISTRY

(Choice B) Ultraviolet radiation causes the formation of pyrimidine-pyrimidine


dimers, not purine-purine dimers.
(Choice C) Deamination of bases in DNA can occur spontaneously or secondary to
exposure to certain chemicals. For instance, cytosine is converted to uracil by
spontaneous deamination. Adenine and guanine can also be deaminated to xanthine
and hypoxanthine, respectively. Removal of abnormal bases such as these and
replacement with the correct base is accomplished by the process of base excision
repair. In base excision repair, abnormal bases are recognized and removed by
specific glycosylase without disruption of the phosphodiester backbone. This removal
produces apurinic and apyridimidinic sites that are cleaved by specific nucleases. The
gap is then filled in by DNA polymerase and joined by ligase.
(Choice D) During DNA replication, incorrect base substitution can occur; for
instance, cytosine could be inserted instead of thymidine. Specific proteins scan the
newly synthesized DNA strands looking for mismatched bases. This process is guided
by the presence of adenine methylation in the template strand and the absence of
methylation in the newly synthesized strand. This difference allows the repair
mechanism to differentiate the newly synthesized DNA strand (no methylation) from
the template strand.
Educational Objective:
After UV damage, pyrimidine dimers are formed in cellular DNA, which are
recognized by a specific endonuclease which initiates the process of repair by nicking
the strand at the thymine dimmer. This action signals the removal and replacement
of this damaged DNA.

255

USMLE WORLD STEP 1

BIOCHEMISTRY

Q NO 43: The liver converts a variety of pyruvate-forming substances to glucose


during starvation. Which of the following is an allosteric activator of this process?
A. Acet-CoA
B. Alanine
C. Citrate
D. Oxaloacetate
E. Lactate
F. Fructose 2, 6-bisphosphate

256

USMLE WORLD STEP 1

BIOCHEMISTRY

Explanation:
Two major processes maintain plasma glucose concentrations within normal limits
between meals: glycogenolysis and gluconeogenesis. Glycogenolysis can maintain
blood glucose levels for only a few hours of fasting until glycogen stores are
depleted. Subsequently, gluconeogenesis is the predominant method used by the
body to maintain blood glucose concentrations.
In gluconeogenesis, glucose is formed from lactate, glycerol, and glucogenic amino
acids. Pyruvate cannot be converted to phosphoenolpyruvate because pyruvate
kinase is unidirectional. To convert pyruvate to phosphoenolpyruvate, pyruvate is
first converted to oxaloacetate by biotin-dependent carbamylation of pyruvate to
oxaloacetate in the mitochondria. Oxaloacetate is converted to malate by malate
dehydrogenase to shuttle out of the mitochondria and is converted again to
oxaloacetate by cytosolic malate dehydrogenase. The activity of pyruvate
carboxylase is increased by acetyl C0A (choice A).
(Choice B) Muscle converts pyruvate to alanine by transamination, and alanine is
then transported to the liver where it is converted back to pyruvate for use in
gluconeogenesis. Alanine allosterically inhibits pyruvate kinase. It thereby inhibits
glycolysis but does not promote gluconeogenesis.
(Choice C) Citrate is formed within mitochondria in the first reaction of the TCA
cycle, and it is also used in the cytosol for fatty acid, ketone body and cholesterol
synthesis. Citrate is an important regulator of acetyl CoA carboxylase and
phosphofructokinase- key enzymes in fatty acid synthesis and glycolysis
respectively.
(Choice D) Oxaloacetate formed in the cytosol is then converted to
phosphoenolpyruvate by the enzyme phosphoenolpyruvate carboxykinase (PEPCR).
Two other enzymes that are different in gluconeogenesis are fructose 1, 6
bisphosphatase and glucose-6-phosphatase.
(Choice E) Lactate is a predominant source of carbon atoms for glucose synthesis by
gluconeogenesis. During anaerobic glycolysis in skeletal muscle, pyruvate is reduced
to lactate by lactate dehydrogenase (LDH). Lactate formed by glycolysis in the
contracting muscles is released into the blood stream and transported to the liver
where it is converted to glucose by gluconeogenesis.
(Choice F) Regulation of phosphofructokinase- and fructose 1, 6-bisphosphatase is
the most significant site for regulating glycolysis and gluconeogenesis. Fructose 26bisphosphate is a powerful negative allosteric effector of fructose 1, 6bisphosphatase activity, one of the enzymes involved in gluconeogenesis.
Phosphatase activity will decrease fructose 2, 6-bisphosphate and will accelerate
gluconeogenesis while kinase activity will increase fructose 2, 6-bisphosphate levels
and accelerate glycolysis.
Educational Objective:
Acetyl-CoA is an important allosteric activator of gluconeogenesis that acts by
increasing the activity of pyruvate carboxylase.

257

USMLE WORLD STEP 1

BIOCHEMISTRY

Q NO 44: Cultured fibroblasts taken from an infant suffering from hypotonia and
seizures show an impaired ability to oxidize yew long chain fatty acids (VLCFA)
and phytanic acid. The defect is most likely localized to:
A. Mitochondria
B. Rough endoplasmic reticulum
C. Proteasomes
D. Lysosomes
E. Peroxisomes
F. Golgi apparatus
Explanation:
The patient described in the question stem is most likely suffering from a
peroxisomal disease. Peroxisomal diseases are rare inborn errors of metabolism
where peroxisomes are either absent or nonfunctional. Very long chain and some
branched chain fatty acids cannot undergo mitochondrial beta-oxidation. These fatty
acids are metabolized by a special form of beta oxidation (very long chain fatty
acids) or by alpha oxidation (branched chain fatty acids such as phytanic acid) within
peroxisomes. When peroxisomes are absent or nonfunctional, these fatty acids
accumulate within the tissues. One example of a peroxisomal disease is Zellweger
syndrome. In this condition, infants are unable to properly form myelin in the CNS.
Symptoms of this disease include hypotonia and seizures as mentioned in the
question stem as well as hepatomegaly, mental retardation, and early death within
months of initial presentation. Refsum disease results from a defect in peroxisomal
alpha oxidation and leads to neurologic disturbances in response to accumulation of
phytanic acid within the body. Treatment of this disease is by strict avoidance of
chlorophyll in the diet.
(Choice A) The mitochondria is the primary site of beta oxidation, ketogenesis, the
TCA cycle, the electron transport chain, the initial and final steps of the urea cycle,
and of decarboxylation reactions among others. It does not metabolize very long
chain fatty acids or fatty acids with branch points at odd-numbered carbons.
(Choice B) The rough endoplasmic reticulum is the site of synthesis of proteins
destined for organelles, cell membrane proteins and extracellular proteins. Proteins
synthesized in the RER are sent to the Golgi apparatus for sorting.
(Choice C) Proteasomes function to degrade unneeded or improperly formed
intracellular proteins to small polypeptides or to amino acids. The proteasome is
essential for regulation of cellular processes because it degrades proteins that
express a function that is no longer needed by the cell. Proteasomes also function to
degrade viral proteins for expression on MHC Class I molecules for recognition byT
lymphocytes.
(Choice D) Lysosomes are organelles containing an acidic fluid with various proteins
for degrading fatty acids, carbohydrates, proteins and nucleic acids. Diseases
resulting from lysosomal dysfunction include the mucopolysaccharidoses most
classically among others.
(Choice F) The Golgi apparatus serves to sort proteins from the rough ER and route
them to their ultimate location within membrane-bound vesicles.
Educational Objective:
Peroxisomal diseases are rare inborn errors of metabolism where peroxisomes are
either absent or nonfunctional. Very long chain fatty acids or fatty acids with branch
points at odd-numbered carbons can not undergo mitochondrial beta-oxidation;
these fatty acids are metabolized by a special form of beta oxidation (very long chain
fatty acids) or by alpha oxidation (branched chain fatty acids such as phytanic acid)

258

USMLE WORLD STEP 1

BIOCHEMISTRY

within peroxisomes. These diseases commonly lead to neurologic defects from


improper CNS myelination.

259

USMLE WORLD STEP 1

BIOCHEMISTRY

Q NO 45: Urea is an essential molecule generated by the body for the disposal of
nitrogen. Which of the following is an immediate source of nitrogen for urea in the
urea cycle?
A. Glutamine
B. Aspartate
C. N-acetylglutamate
D. Asparagine
E. Alanine
Explanation:
Ureas nitrogen is derived from NH3 and aspartate in the urea cycle. The urea cycle
is the biochemical pathway where by toxic nitrogen molecules are converted into
urea for excretion in the kidneys. Below is an outline of the urea cycle:

Carbamoyl phosphate synthetase I (CPS I) is the rate-limiting enzyme in the urea


cycle and is activated by N-acetylglutamate (NAG), which in turn is produced from
glutamate and acetyl CoA in a reaction activated by arginine.
(Choice A) Glutamine, one of the 20 major amino acids, is the foremost amino acid
in the blood because it transports ammonia from the peripheral tissues to the
kidney. In the nephron, the amides nitrogen is hydrolyzed by the enzyme
glutaminase, thereby producing glutamate and free ammonium ion. The ammonium
ion can then be excreted in the urine.
(Choice C) N-ace glutamate synthetase (NAGS) is confined to the hepatic
mitochondria and mediates the following reaction:
Acetyl-coenzyme A (C0A) + glutamate = N-acetylglutamate + Co
The normal function of N-acetylglutamate (NAG) is to activate carbamoyl phosphate
synthetase I (CPSI), another mitochondrial enzyme. Remember that CPSI catalyzes
the rate-limiting step in the urea cycle by producing carbamoyl phosphate within the
mitochondria, which then combines with ornithine to produce citrulline.
(Choice D) Asparagine is a non-essential amino acid with a nitrogen-containing side
chain that is produced from oxaloacetate in a transamination reaction. Oxaloacetate
is first converted to aspartate using glutamate to provide the amino group; then, the

260

USMLE WORLD STEP 1

BIOCHEMISTRY

enzyme asparagine synthase transfers the additional nitrogen molecule from


glutamine to aspartate to produce asparagine.
(Choice E) Alanine is a nonpolar amino acid that is most commonly produced by
transfer of an amino group to pyruvate; it has a single methyl group as its side
chain. Alanine does not play a prominent role in the urea cycle.
Educational Objective:
The nitrogen atoms in the urea molecule are derived from NH3 and aspartate in the
urea cycle. Remember that carbamoyl phosphate synthetase I (CPS I) is the ratelimiting enzyme in the urea cycle reaction and is activated by N-acetylglutamate
(NAG).

261

USMLE WORLD STEP 1

BIOCHEMISTRY

Q NO 46: A group of investigators studies the mechanism of catecholamine


synthesis regulation. In animal experiments, hypophysectomy induced a
decrease in epinephrine secretion by the adrenals. Decreased activity of
which of the following enzymes is most likely responsible for the effect
observed?
Phenylalanine hydroxylase
B. Tyrosine hydroxylase
C. Dopa decarboxylase
D. Dopamine beta-hydroxylase
E. Phenylethanolamine-N-methyltransferase
F. Catechol-O-methyl transferase
G. Monoamine oxydase (MAO)

A.

Explanation:
The three main circulating catecholamines are dopamine, norepinephrine, and
epinephrine. In contrast to norepinephrine and dopamine, epinephrine is chiefly
produced by the adrenal glands. Norepinephrine and dopamine are produced by both
the central and peripheral nervous system. Dopamine is norepinephrines precursor.
In the adrenal medulla, the first step in the synthesis of catecholamine (see
flowchart) is conversion of tyrosine to dihydroxyphenylalanine, more commonly
called dopa, by the enzyme tyrosine hydroxylase. Tyrosine is derived either from
ingested food or from phenylalanine synthesis in the liver. The conversion of
dihydroxyphenylalanine is accomplished by phenylalanine hydroxylase. Dopa, then,
is converted to dopamine by dopa decarboxylase, which is then converted to
norepinephrine by dopamine -hydroxylase.
The conversion of norepinephrine to epinephrine occurs mostly in the adrenal
medulla by an enzyme called Phenylethanolamine-N-methyltransferase (PNMT).
Cortisol increases the transcription of this enzyme. Because venous drainage of the
adrenal cortex goes through the adrenal medulla the concentration of cortisol is very

262

USMLE WORLD STEP 1

BIOCHEMISTRY

high in the adrenal medulla. Cortisol increases expression of the gene encoding
PNMT. The catecholamine contents of the normal human adrenal medulla are
approximately 80% epinephrine and 20% as norepinephrine. This epinephrine heavy
ratio is due to the positive effect of cortisol on the expression of the enzyme PNMT,
which converts norepinephrine to epinephrine.
(Choices F and G) Catechol-O-methyltransferase (COMT) and monoamine oxidase
(MAO) are the enzymes responsible for metabolism of catecholamines. MAO converts
epinephrine and norepinephrine to dihydroxymandelic acid. COMT converts
epinephrine to metanephrine and converts norepinephrine to normetanephrine.
Educational Objective:
Phenylethanolamine-N-methyltransferase (PNMT) I which is responsible for the
synthesis of epinephrine, is under the control of cortisol.

263

USMLE WORLD STEP 1

BIOCHEMISTRY

Q NO 47: E. coil are grown on a medium containing lactose. Once glucose is added
to the medium, the bacteria stop fermenting lactose. Which of the following best
explains the observed effect?
A. Repressor protein is bound to the operator
B. Repressor protein is bound to the promoter
C. Repressor protein is bound to glucose
D. Glucose is bound to the promoter
G. Cellular level of cAMP is low

264

USMLE WORLD STEP 1

BIOCHEMISTRY

Explanation:
The initiation of transcription in prokaryotes is controlled by two upstream DNA
sequences, known as promoter sequences, which promote transcription by RNA
polymerase. There are a number of accessory proteins that regulate the interaction
of RNA promoters at the transcription site. These proteins can work as activators or
repressors of gene transcription. Operon is the term used to describe a DNA
sequence that contains at least two regulatory sites, the promoter and the regulator
(operator) in addition to the gene coding for the structure of the protein itself.
The fenopron is the genetic sequence in the E. coil genome that codes for proteins
required for the metabolism of lactose. The lac operon consists of a regulatory gene
(I), promoter region (p) operator region (o), and three structural genes (z, y and a).
The z gene codes for -galactosidase (-gal) which is primarily responsible for the
hydrolysis of lactose to glucose and galactose. The y gene codes for permease a
transmembrane enzyme that increases the permeability of the cell to lactose. The I
gene codes for the repressor protein of the lac operon, which binds the DNA
sequence at the operator to prevent binding of RNA polymerase to the promoter. The
p gene is the promoter region, the initial binding site for RNA polymerase, to initiate
transcription of the lac operon.
Culturing E. coil in lactose-containing media results in the binding of lactose to the
repressor protein. This binding causes a conformational change which prevents the
attachment of the repressor protein to the operator region which in turn increases
transcription of the lac operon structural genes and, subsequently increases
utilization of lactose for energy. Culturing E. coil in glucose-containing media with or
without lactose represses the expression of the lac operon. Glucose decreases the
activity of adenylyl cyclase and leads to a reduction in intracellular cAMP. Depletion
of cAMP by the presence of glucose decreases the expression of the lac operon
structural genes. High cAMP levels activate a protein called catabolite activator
protein (CAP) which binds cAMP to form a cAMP-CAP complex which then binds to a
region upstream from the promoter region and acts as positive regulator of the lacoperon. The main mechanism of glucose-induced inhibition of lac operon expression
is depletion of cAMP (Choice E) not binding of the repressor protein to the operator
(Choice A).
(Choice B) Repressor proteins do not bind to promoter region: they bind the
operator region.
(Choices C and D) In contrast to lactose which acts as an inducer glucose does not
bind to the repressor protein or promoter.
Educational Objective:
Glucose induced decreased adenylate cyclase activity leads to low intracellular
concentrations of cAMP. Low cAMP levels in turn cause poor binding of catabolite
activator protein (CAP) to the CAP-DNA binding domain leading to decreased
expression of the structural genes of the lac operon.

265

USMLE WORLD STEP 1

BIOCHEMISTRY

Q NO 48: A 6-month-old male is brought to the ER with recent on set vomiting,


irritability and jaundice. The infant was born at term and has been apparently
healthy until the onset of these symptoms. All of his vaccinations are up-todate. He has been exclusively breast1ed until one week ago when cereals and
fruit juices were introduced into his diet. Further evaluation reveals
hepatomegaly and abnormal liver function tests. Which of the following enzyme
deficiencies is the most likely cause of this patients condition?
Galactose-1-phosphate uridyl transferase
B. Aldolase B
C. Fructokinase
D. Galactokinase
E. Acid a-glucosidase
Explanation:
Dietary fructose is obtained mainly from fruits, vegetables, honey and table sugar
(sucrose). Fructose is rapidly absorbed in the proximal small bowel by the hexose
transporter GLUT 5. Initial metabolism of fructose involves three enzymes namely
fructokinase, aldolase B and triokinase. Fructose is phosphorylated on the first
carbon by hepatic fructokinase yielding fructose-i-phosphate (FIP). Metabolism of
fructose-i-phosphate by aldolase B generates dihydro acetone phosphate (DHAP)
and glyceraldehyde. DHAP is converted by triose phosphate isomerase to
glyceraldehyde- -phosphate, an intermediate of glycolysis. Glyceraldehyde can be
phosphorylated to GSP by triose kinase or converted to DHAP.

266

A.

USMLE WORLD STEP 1

BIOCHEMISTRY

Aldolase deficiency causes the potentially life-threatening disorder hereditary


fructose intolerance. The clinical symptoms of this disease can be minimized by
elimination of dietary fructose. Patients typically present when fructose-containing
foods are introduced into the diet. The primary manifestations are vomiting and
hypoglycemia about 20 to 30 minutes after fructose ingestion. These infants present
with failure to thrive, hepatomegaly, and cirrhosis. Because similar clinical features
may be present with galactosemia or tyrosinemia, the diagnosis may be confirmed
by genetic testing.
(Choices A and D) Galactosemia is an autosomal recessive disorder characterized by
neonatal jaundice. Bleeding diathesis, feeding intolerance, hypoglycemia, mental
retardation hypotension and death if untreated. Symptoms start soon after breast
bleeding is initiated. This clinical syndrome is usually caused by defective galactose1-phosphate uridyl transferase (classic galactosemia) and results in the
accumulation of galactose in the blood and excretion of excess galactose in the
urine. Treatment is with discontinuation of all milk products and feeding with a soybased infant formula. Galactokinase deficiency causes increased serum galactose
concentration, liver enlargement, cataracts and mental retardation.
(Choice C) Fructokinase deficiency causes essential fructosuria, a rare autosomal
recessive benign asymptomatic disorder. Fructose from diet is absorbed and secreted
in the urine due to defective metabolism. Fructose similar to glucose and galactose,
is a reducing sugar and is detected by tested by Clinitest tablets (which tests
nonspecifically for reducing sugar). A urine dipstick utilizes glucose oxidase method
for determination of urine glucose will not test positive with fructose or galactose.
(Choice E) Alpha glucosidase (or acid maltase) deficiency causes glycogen storage
disease II (Pompe disease). Glycogenolysis (breakdown of glycogen) is accomplished
mainly by glycogen phosphorylase and debranching enzyme, but a small amount of
glycogen is also broken down by the lysosomal enzyme alpha 14- glucosidase. This
disease does not present with hypoglycemia but with hepatomegaly, cardiomyopathy
and increased risk for cirrhosis.
Educational Objective:
Aldolase deficiency causes hereditary fructose intolerance. This disease manifests
after introduction of fructose into the diet with vomiting and hypoglycemia about 20
to 30 minutes after fructose ingestion. These infants present with failure to thrive,
hepatomegaly, and cirrhosis.

267

USMLE WORLD STEP 1

BIOCHEMISTRY

Q NO 49: A colony of cells is exposed to high-dose cortisol in an experimental


setting. Which of the following intracellular substances is most likely to
increase immediately after the exposure?
cAMP
B. IP3
C. Unbound hsp90
D. Activated Ras
E. Phosphorylated JAK
F. mRNA

A.

Explanation:
Receptor proteins are either extracellular (located on the cell surface) or intracellular
(located in the cytoplasm or nucleus). The majority of receptors are extracellular,
with steroid hormone receptors the intracellular exception.
Because steroid hormones are fat-soluble, they can penetrate the cell membrane
without the assistance of a transmembrane channel. Receptors for these steroid
hormones are therefore located in the cell cytoplasm or, occasionally, the nucleus.
The steroid receptor is a zinc finger protein that, while inactive, is bound to heat
shock proteins (specifically, hsp90 and hsp56). Once the steroid hormone makes
contact with its receptor, the heat shock proteins are released into the cytoplasm.
The release of these proteins exposes the DNA-binding area of the receptor, and the
receptor-hormone complex is transported to the nucleus. The transcription of specific
genes is activated by this DNA-binding area.
(Choice A) The cyclic AMP (cAMP) pathway is a signal transduction system that
transmits stimulus from the receptor to the cell nucleus. The receptor is bound to a
G protein that activates adenylyl cyclase, which produces large quantities of cAMP
that bind to and activate protein kinase A. The cAMP pathway is not associated with
steroid receptors.
(Choice B) IP3 is another signaling system. Stimulus of the receptor activates
phospholipase C, which produces inositol triphosphate (IP3) and diacylglycerol
(DAG). IP3 increases the level of calcium in the cytosol, and these calcium ions then
activate protein kinase C. The IP3 pathway is not associated with steroid receptors.
(Choice D) The Ras protein is a component of the MAP-kinase system. It is
associated with tyrosine kinase receptors, but not steroid receptors.
(Choice E) JAK (Janus kinase) is a component of the JAK/STAT signal transducing
pathway. JAK is affiliated with tyrosine-kinase associated receptors, but not steroid
receptors.
(Choice F) mRNA (messenger RNA) carries information from nuclear DNA to the
ribosomal sites of protein synthesis in the cytosol.
Educational Objective:
Steroid receptors are generally located in the cytoplasm. When inactive, they are
bound to heat shock protein 90 (hsp9o) and heat shock protein 56 (hsps6). These
heat shock proteins are released when the receptor binds the steroid hormone, and
the receptor-hormone complex is then transported into the nucleus for gene
transcription.

268

USMLE WORLD STEP 1

BIOCHEMISTRY

Q NO 50: A 46-year-old male is brought to the emergency department by EMS. He


has attempted suicide twice in the past, once by cutting his wrists and another time
by taking an overdose of amitriptyline. His current medications include quetiapine
and fluoxetine. He has no known drug allergies. This evening a neighbor found him
in a closed garage with the car running. As you examine him he loses
consciousness and begins to seize. The toxic substance causing this patients
condition affects hemoglobin by:
A. Oxidation of the iron moiety
B. Oxidation of the porphyrin ring
C. Covalent linking to heme
D. Competitive binding to heme
E. Altering the partial pressure of oxygen
Explanation:
Carbon monoxide (CO) is a colorless, odorless nonirritant gas that is generated as a
byproduct of incomplete hydrocarbon combustion. Carbon monoxide emission from
automobiles can result in carbon monoxide poisoning in poorly ventilated spaces.
The scenario described above is typical for carbon monoxide poisoning. Another
classic source of carbon monoxide poisoning is a faulty home heater.
Carbon monoxide has 220 times more affinity for hemoglobin than does oxygen.
Inhaled carbon monoxide rapidly diffuses across the alveolar membrane and binds
tightly with heme-bound iron in hemoglobin and other hemeproteins. Carbon
monoxide binding to hemoglobin results in formation of carboxyhemoglobin. Carbon
monoxide decreases the oxygen content of the blood by occupying oxygen binding
sites. Carbon monoxide also inhibits the release of oxygen from hemoglobin in
tissues by altering hemoglobin conformation into the relaxed form that has a very
high affinity for oxygen. This results in a leftward shift of the oxygen dissociation
curve and tissue hypoxia via deficient unloading of oxygen. Treatment of carbon
monoxide toxicity is with 100% or hyperbaric oxygen.
(Choice A) Iron bound to heme is in the reduced ferrous (Fe++) state. Oxidation of
ferrous iron to ferric iron (Fe+++) leads to the formation of methemoglobin.
Hemoglobin containing ferric iron (methemoglobin) is unable to bind to oxygen.
(Choices B and C) Heme is an iron containing protoporphyrin IX, where ferrous iron
is held in the center of heme molecule by four nitrogen in the porphyrin ring. Carbon
monoxide binds to iron in metalloproteins, but it does not alter porphyrin oxidation
in cytochrome oxidase or covalent linkage in cytochrome C.
(Choice E) The partial pressure of oxygen (p02) is dependent on the concentration of
oxygen dissolved in the plasma and not on actual concentration or content of oxygen
in hemoglobin. The amount of oxygen dissolved in plasma does not change in CO
poisoning, so the p02 is not decreased.
Educational Objective:
CO binds to hemoglobin with an affinity that is 220 times that of oxygen for
hemoglobin. The binding of CO and O2 to hemoglobin are reversible. CO, therefore,
competes with O2 for binding on the heme iron of hemoglobin.

269

You might also like